Download as pdf or txt
Download as pdf or txt
You are on page 1of 157

‫هذا الملف هو نتاج جهد متواصل لحل أسئلة الجراحة العامة‪ ،‬أسال هللا أن يجعله علما ً نافعا ً وعمالً

متقبالً‪.‬‬

‫متمنيا ً لكم دوام التوفيق والسداد‪.‬‬


Trauma + ATLS
Q1- A young man was beaten many times on the face. A trial of endotracheal intubation has
failed.
How will you secure the airway?

A - Tracheostomy tube
B - Cricothyroidotomy
C - Orotracheal tube
D - Nasopharyngeal airway

Answer: B
Surgical Cricothyroidotomy.

_____________________________________

Q2 - Trauma with tension pneumothorax. GCS 8.


What to do next?

A - Intubation
B - Needle thoracostomy

Answer: A
Although in real life trauma management various interventions are done simultaneously (e.g.,
while someone is busy intubating another person is establishing an IV line and a third is placing
a splint to stabilize a broken limb), in exam setting, you can’t move from one step to another
before you have addressed it fully.
So, you can’t move to B before you finish A, no matter how bad B is compromised, and so on.

So here, A takes precedence over B.


Someone with GCS of 8 has a clear indication for intubation and that is what you start with.

_____________________________________

Q3 - Adult fell from height complains of severe heel pain. He is conscious, oriented and has
stable vitals.
What is the next step?

A - Pain control
B - Lower limb Xray
C - Pulse palpation

Answer: C
All those will need to be done. But first things come first. You need to complete your physical
examination of the affected limb by doing a quick neuro-vascular assessment.
_____________________________________

1
Q4 - Deep wound in anterior thigh,10 cm long, actively bleeding.
How to control bleeding?

A - Direct pressure on the wound


B - Apply torniquet
C - Pressure on the femoral vessel above the wound

Answer: A
Direct pressure on the wound.

_____________________________________

Q5 - Male victim of gunshot to the thigh, he is pale and unconscious. BP 90/60 pulse 130.
What is the next step in management?

A - Orotracheal intubation
B - Blood transfusion
C - Shift to OR
D - LR infusion

Answer: A
Although this patient has a clear problem with “C”, he is also unconscious and will not be able to
protect his airway.
“A” takes precedence over “C” So the answer is: Endotracheal intubation.

_____________________________________

Q6 - Chest gunshot wound. Entry was lateral to left nipple and exit point below left scapula. The
patient suddenly deteriorated with raised JVP and muffled heart sounds.
What is your action?

A - Pericardiocentesis
B - Immediate thoracotomy
C - CT

Answer: A
Cardiac tamponade, pericardiocentesis.

_____________________________________

2
Q7 - Patient involved in MVA sustained complex femoral fracture and tension pneumothorax.
Chest tube was inserted and patient was transferred to a higher center. In transit, the patient
desated and became tachypnic and tachycardic.
What to do?

A - Intubation
B - Nothing
C - Check for bleeding form fracture site
D - Confirm placement of chest tube

Answer: D
As a rule in trauma management, whenever patient’s condition changes, you repeat your primary
survey (ABCDE) again.

This is not a complete scenario; we don’t know if chest tube is working or not (bubbling and / or
fluctuating underwater seal). We don’t know about chest exam (decreased air entry?!!
Hyperresonant percussion note?!!).
We don’t know if trachea is shifted or not.
With the limited data given, I will go for D.

_____________________________________

Q8 - A male victim of MVA presents to ER with SOB. On examination, trachea is shifted to the
right side. Chest X-ray, both lungs are expanded and. And mediastinum is widened.
What is the diagnosis?

A - Tension pneumothorax
B - Cardiac tamponade
C - Plural effusion
D - Raptured esophagus

Answer:
Can’t be A or C because lungs are expanded.
B will not cause widened mediastinum.

We are left with D. Esophageal perforation will cause widening of mediastinum, but why would
it cause SOB if there is no effusion (lungs are expanded)

If the question comes this way, go for D with a bunch of salt!!

_____________________________________

3
Q9 - A 35-year-old car unrestrained driver crashed into a concrete block. On the way to the
hospital, he started to develop shortness of breath in the ambulance. 100% oxygen was
administered with not much improvement in this condition. On arrival to Emergency
Department, he has lost consciousness and appears cyanosed with markedly distended jugular
veins.
Blood pressure 80/40
Heart rate 120 /min
Respiratory rate 34 /min
Temperature 36.6 c
Oxygen saturation 60% on room air
What immediate action should be taken?

A - Intubation and 100% oxygen


B - Rapid infusion of crystalloid
C - Needle decompression
D - IV 0.2 mg adrenaline

Answer: C
Tension pneumothorax is caused by pressure accumulating within one side of the thorax. This
results in one lung collapsing on the affected side and placing pressure on the mediastinum,
which compromises blood flow. The collapsed lung causes diminished breath sounds because the
lung is collapsed and no longer expanding. Pressure on the mediastinum creates back pressure on
the heart, causing jugular vein distention (JVD). Without relief of this pressure via thoracic
needle decompression, the patient’s condition will continue to deteriorate.

_____________________________________

Q10 - Trauma patient, hypotensive with Hb of 8.


What to do?

A - IV normal saline.
B - Blood transfusion.

Answer:
ATLS protocol emphasizes the importance of restricting IVF and considering early blood
transfusion in management of trauma patients.

The algorithm goes this way:


Start with one liter of crystalloids, if no response, don’t give any more IVF, go directly to blood
transfusion (uncross matched till cross-matched blood arrives).

This is not a realistic question anyway. One can’t imagine that we already have our lab results
back for a patient in hypovolemic shock, but we are still debating what the initial fluid should be.
_____________________________________

4
Q11 - Victim of RTA complains of dyspnea and chest pain. On examination, he is conscious and
oriented. Chest exam shows decreased air entry in the left side with hyper-resonant note.
Abdominopelvic exam was normal. Vitals, SpO2 90% other vitals within normal limits.
What is the best next step in management?

A - Plain x Ray
B - Needle decompression
C - Chest tube

Answer: A
Simple pneumothorax
Chest X-ray confirmation followed by chest tube insertion in the fifth left intercostal space.

_____________________________________

Q12 - Victim of RTA presents with facial and basal skull fractures.
What is the first step in management?

A - Support the airway


B - Support the neck

Answer:
ATLS is clear about that. “Secure airway with c-spine immobilization”. They go hand to hand.
It is really odd to have to choose between them. If both options appear in the question, go for
airway.

_____________________________________

Q13 - Victim of RTA, stable everything is fine no clear fractures, suddenly deteriorates.
What is the best way to secure airway?

A - Nasotracheal tube
B - Orotracheal tube

Answer: B
Incomplete scenario.

_____________________________________

5
Q14 - Man after accident and resuscitation in a small hospital. You need to transfer to another
hospital after stabilization, it is 40 mins away. On x-ray you see fracture of 2 to 5 left ribs. No
pneumothorax.
What will u do?

A - Intubate
B - Call the other hospital to inform the surgeon on call
C - Chest tube insertion
D - DPL

Answer: B
Clearly, there is no indication for intubation nor DPL.
No hemo or pneumothorax to indicate chest tube insertion.
As per ATLS protocol, call the receiving hospital and discuss the case with the on-call surgeon.

_____________________________________

Q15 - Burn patient, resuscitation done.


Which of the following reflect good resuscitation?

A - Normalization of heart rate


B - Normalization of blood pressure
C - Urine out of 0.6ml/kg/h
D - Central venous pressure 12

Answer: C
In contrast to resuscitation for other types of traumas in which fluid deficit is typically secondary
to hemorrhagic losses, burn resuscitation is required to replace the ongoing losses from capillary
leak due to inflammation.

Blood pressure measurements can be difficult to obtain and may be unreliable in patients with
severe burn injuries. Insert an indwelling urinary catheter in all patients receiving burn
resuscitation fluids and monitor urine output to assess perfusion.

The actual fluid rate that a patient requires depends on the severity of injury, because larger and
deeper burns require proportionately more fluid. If the initial resuscitation rate fails to produce
the target urine output, increase the fluid rate until the urine output goal is met.

It is important to understand that under-resuscitation results in hypoperfusion and end organ


injury. Over- resuscitation results in increased edema, which can lead to complications, such as
burn depth progression or abdominal and extremity compartment syndrome. The goal of
resuscitation is to maintain the fine balance of adequate perfusion as indicated by urine output.

_____________________________________

6
Q16 - 10 cm thigh wound with profuse bleeding.
How to control it?

A - Apply pressure on wound


B - Apply pressure on femoral artery

Answer: A

_____________________________________

Q17 - Patient with stab wound to anterior neck. He is alert. Oxygen sat 82%.
What to do?

A - Oxygen mask
B - Cricothyroidotomy
C - Endotracheal intubation
D - Tracheostomy

Answer: C
Penetrating neck wounds puts airway in jeopardy because of compression by expanding
hematoma.
Endotracheal intubation

_____________________________________

Q18 - Women had MVA. She has a tender abdomen and ecchymosis, but there’s no peritonitis
signs. Her BP is fine.
What is the next initial step?

A - IV crystalloid
B - FAST
C - Surgical exploration

Answer:
The “next initial” step is IVF
Remember ATLS approach.
-IVF comes with the C of primary survey.
-FAST is an adjunct to primary survey.
-Exploratory laparotomy is a definitive therapy.

_____________________________________

7
Q19 - Case of MVA. Hemodynamically stable. FAST showed perihepatic and peri-splenic free
fluids.
What is the most appropriate management?

A - Exploratory laparotomy
B - CT

Answer: B
In blunt abdominal trauma, decision to operate is based on hemodynamic stability and not on the
mere presence of blood in peripheral cavity.

_____________________________________

Q20 - Trauma patient with T6 vertebral fracture. He is paraplegic, breathes comfortably, BP 80


What is the most appropriate next step?

A - Elective intubation
B - Normal Saline

- I doubt that RL isn’t in the options.

Answer: A and B are secured “for now at least”. Go for C.


Patient needs volume expansion with crystalloid IVF.
For small amount IVF, NS and LR are basically the same. For excessive volume resuscitation,
LR is superior.

_____________________________________

Q21 - Patient with neck injury, conscious and alert with oxygen saturation of 82%
What is the most appropriate management?

A - Oxygen via face mask


B - Cricothyroidotomy
C - Intubation

Answer: C
Priority goes to securing the airway. Soft tissue injury and expanding neck hematoma puts
airway in jeopardy.

_____________________________________

8
Q22 - Post RTA, he is shouting, and says that he has extreme chest pain.
and he has blood pressure 90.
What to do next?

A - Establish an IV line
B - Do FAST
C - Pericardiocentesis

Answer:
With these limited data… I would go for A

+Why not C?
+ Why do we need to IMAGINE things that are not there.
The only piece of information in the question is chest pain.
This can be caused by a variety of injuries (rib fracture, sternal fracture, clavicular fracture, blunt
cardiac injury, MI, aortic dissection) and the list goes on.
Pericardiocentesis would be shooting in the dark.

_____________________________________

Q23 - Which of the following procedures provides an emergency non-surgical airway when
endotracheal intubation and mask ventilation are not possible?

A - Laryngeal mask
B - Cricothyroidotomy
C - Transtracheal jet ventilation
D - Non-invasive positive pressure ventilation

Answer:
B is surgical
D is not an airway
Both A and C are correct.
But since this is SMLE exam, no one expects you to know transtracheal jet ventilation.
The answer you need to pick is “A”
LMA is an adjunct to airway as detailed in Airway Decision Scheme.

_____________________________________

9
Q24 - RTA, sternal fracture, decreased heart sounds, and hypotension.
What is the most likely diagnosis?

Answer:
Cardiac Tamponade.

_____________________________________

Q25 - MVA patient with paradoxical chest movement. Oxygen sat: 80%.
What is the next step in management?

A - IV fluid
B - Needle decompression
C - Assisted ventilation
D - Chest tube insertion

Answer: C

_____________________________________

Q26 - Patient with blunt chest injury resulted in fracture in 3rd, 4th and
5th ribs in more than one site.
What is the initial treatment?

A - Intubation
B - Assisted ventilation
C - IV fluid

Answer: B
Flail chest
Intubation and assisted ventilation are basically the same option (intubation is insertion of
endotracheal tube. For this to have any benefit, it must be connected to a ventilator- assisted
breathing).

_____________________________________

10
Q27 - Patient presents with stab wound to the abdomen. After wound exploration, you found
anterior abdominal fascia penetration. His vitals were stable.
What is your next step?

A - CT abdomen
B - MRI abdomen
C - Exploratory laparotomy
D - Diagnostic laparoscopy

Answer:
Basically, anything that is done open can be done laparoscopically.
Fascial penetration is an indication for surgery.

In a hemodynamically stable patient, both open and laparoscopic exploration are valid options. It
all comes down to the expertise of the treating surgeon and how comfortable he or she is with
laparoscopy.

Laparoscopy is contraindicated in hemodynamically unstable patients as pneumoperitoneum may


further compromise hemodynamics by compressing IVC, hence, decreasing venous return.

_____________________________________

Q28 - A patient involved in high-speed RTA. Abdominal examination shows diffuse tenderness
and guarding.
What will you do next?

A - Emergent exploratory laparotomy


B - FAST
C - CT abdomen

Answer: A

General role in surgery.


An existing indication for surgery is a contraindication for further imaging
This patient has a surgical abdomen and the only place he should be taken to is OR.

_____________________________________

11
Q29 - A 22-year-old sustains a stab to the anterior abdomen. He has stable hemodynamics with
no shock, no evisceration. He has tenderness around the stab wound.
Which of the followings is the most appropriate initial management?

A - CT or DPL
B - Exploratory laparotomy
C - Local wound Exploration
D - Serial abdominal examination

Answer:
The data provided in the question don’t pose an indication for laparotomy.

Serial abdominal examination is an important component in continuing follow up after a


“negative CT, as small bowel injuries can be easily missed by early CT and would only show by
serial abdominal exam

Wound exploration is falling out of favor because of its inaccuracy .

The answer is CT scan.

_____________________________________

Q30 - Victim of MVA received resuscitation was then deteriorated with BP of 90/60. What is the
next step?

A - FAST
B - Exploratory Laprotmy

Answer: Incomplete question


As per the information provided, we have no reason to assume that abdomen is the source of
bleeding

In trauma patients, any change in clinical status would mandate repeating primary survey and its
adjuncts.

If these are the only two options, go for FAST.


_____________________________________

12
Q31 - Patient presents with left flail chest but stable. After that he deteriorated. CXR showed left
defined lung opacity.
What is the diagnosis?

A - Flail chest
B - Pulmonary contusion

Answer: B
The structure of the question is not right.
They can’t tell you it is a flail chest then ask you whether it is a flail chest or not!!

When a force that is significant enough to cause rib fracture is transmitted to the underlying lung
tissues, pulmonary contusion develops.
_____________________________________

Q32 - Victim of MVA presents with paradoxical chest movement. Oxygen sat: 80%.
What is the most appropriate management?

A - IV fluid
B - Needle decompression
C - Assisted ventilation

Answer: C
Flail chest describes a situation in which a portion of the rib cage is separated from the rest of the
chest wall, usually due to a severe blunt trauma, such as a serious fall or a car accident. This
affected portion is unable to contribute to expansion of the lungs. Treatment focuses on
ameliorating the effect of flail segment on ventilation dysfunction through pain medications and
assisted ventilation.
_____________________________________

Q33 - Patient with blunt chest injury resulted in fracture of 3rd, 4th and
5th ribs in more than one site.
What is the initial treatment?

A - Intubation
B - Assisted ventilation
C - IV fluid.

Answer:
Intubation and assisted ventilation go hand to hand with each other. Intubation per se doesn’t
address the problem of ventilation. You got to connect the patient to a ventilator.

_____________________________________

13
Q34 - Trauma patient in ER after you finish primary survey, his BP is 90/50
What is the most appropriate next step?

A - IV fluid
B - Call surgeon in duty

Answer: A

_____________________________________

Q35 - Base of skull fracture passing through the jugular foreman.


Which of the following is expected to be affected?

A - Tongue.
B - Mastication muscle.
C - Ipsilateral vocal cord

Answer: C
A: it gives sensory supply to posterior third of tongue through IX.
B: mastication muscles are not affected
C: vocal cord paralysis because paralysis of laryngeal muscle by IX and loss of gag reflex by X

The answer is C because the sensory effect on tongue always passes unnoticeable.

_____________________________________

Q36 - Trauma patient with positive FAST scan what to do?

A - CT scan
B - Laparotomy

Answer:
Stable, CT
Unstable, Explore
Positive FAST - per se - is not an indication for exploration. It all depends on hemodynamic
stability

_____________________________________

14
Q37 - Male patient with an abdominal stab wound near the umbilicus. He is stable. On wound
exploration, omentum was seen. CT was done and the report literally said (normal)
What is the most appropriate management?

A - Observation
B - Close the wound
C - Leave the wound open
D - Exploratory laparotomy

Answer: D
A penetrating abdominal injury with evisceration of omentum should prompt an exploratory
laparotomy.
In this context, CT is contraindicated.
A negative CT - if done - should not change the decision for exploration, as small bowel injuries
are typically missed by early CT.

It is true that there is a up to 16% rate of negative laparotomies, but that should not change the
approach as the consequences of missing concomitant visceral injuries outweigh those of
negative exploration.

_____________________________________

Q38 - Old patient presents with subdural hematoma with signs of lateralization. GCS is 7/15.
The patient was intubated and resuscitated. Imaging revealed 13 mm midline shift.
What to do next?

A - IV Mannitol
B - Admit ICU and observe
C - Craniotomy
D - Insert intracranial monitor

Answer:
It all depends on how you interpret (next).
Initial... Mannitol.
Definitive... craniotomy.

_____________________________________

15
Q39 - Patient sustains gun shut to right chest. Chest tube (ICT) inserted after 1 hour the ICT is
full of blood, the amount was 200 ml.
What to do next?

A - Insertion of another chest tube


B - Thoracotomy
C - Observation
D - Imaging

Answer: C
Indications for thoracotomy here would be initial output of 1500 cc or 200 cc per hour for three
consecutive hours.

So, the answer here is to continue non operative management


_____________________________________

Q40 - Trauma patient has abdominal wall ecchymosis and left hypochondrium pain. He is stable.
What is the most appropriate investigation?

A - Fast
B - CT

Answer:
FAST is an adjunct to primary survey. It will show you free fluid in the abdomen, but it can’t tell
you it’s source and obviously can’t grade the injury.

CT is the diagnostic modality of choice in a stable trauma patient


_____________________________________

16
Q41 - Patient fell from 4 m height and states that he can’t feel his lower extremities. On
examination, he has warm extremities. He was given crystalloid fluids which improved his BP.
CVP was 2 mmHg BP 76/43 (not sure).
(Other vitals were given but can’t recall).
Which of the following is the best initial management?

A - IV fluid
B - Blood transfusion
C - IV mannitol
D - IV steroid

Answer: A
Neurogenic shock results in the loss of vasomotor tone and sympathetic innervation to the heart.
Injury to the cervical or upper thoracic spinal cord (T6 and above) can cause impairment of the
descending sympathetic pathways. The resultant loss of vasomotor tone causes vasodilation of
visceral and peripheral blood vessels, pooling of blood, and, consequently, hypotension. Loss of
sympathetic innervation to the heart can cause bradycardia or at least the inability to mount a
tachycardic response to hypovolemia. However, when shock is present, it is still necessary to
rule out other sources because hypovolemic (hemorrhagic) shock is the most common type of
shock in trauma patients and can be present in addition to neurogenic shock. The physiologic
effects of neurogenic shock are not reversed with fluid resuscitation alone, and massive
resuscitation can result in fluid overload and/ or pulmonary edema. Judicious use of vasopressors
may be required after moderate volume replacement, and atropine may be used to counteract
hemodynamically significant bradycardia.

_____________________________________

17
Q42 - RTA case asking about most common finding in FAST?

A - Hollow viscus injury.


B - Free intraperitoneal fluid.

Answer: B
_____________________________________

Q43 - Patient post MVA. CXR shows multiple left sided rib from 3 to 7. After some time,
another CXR was done showing left sided basilar atelectasis and effusion. Which of the
following is the most likely diagnosis?

A - Flail Chest
B - Pulmonary Contusion
C - Thoracic Aortic Rupture
D - Pericardial Effusion

Answer: B
Pulmonary Contusion.
_____________________________________

Q44 - Gunshot wound in the thigh, he has weak pulse in the affected limb and
There is hematoma and he can’t move and there’s paresthesia.
What to do?

A - Exploration in the OR
B - Exploration in ER
C - Angiography

Answer: A
_____________________________________

Q45 - Patient involved in high-speed RTA. Abdominal examination shows diffuse tenderness
and guarding.
What will you do next?

A - Emergent exploratory laparotomy


B - FAST
C - CT abdomen

Answer: A
Presence of peritoneal signs (peritonitis) is an indication for exploration.

_____________________________________

18
Q46 - Patient fell from 4 m height, complaining of severe bilateral lower limb pain.
What is the most appropriate next step?

A - Bilateral lower limb x-ray


B - Observation
C - Pain control
D - Check pulse

Answer: D
Checking pule is part of the primary survey, so it comes before the rest of other options.

_____________________________________

Q47 - Young patient presents with a stab wound in the center of the abdomen. He is vitally
stable.
What is the most appropriate next step?

A - FAST
B - DPL
C - CT
D - Exploratory laparotomy

Answer:
In penetrating abdominal trauma, the question is whether fascia is penetrated or not.
That question can’t be answered by FAST
CT would be a reasonable option in absence of surgical indications.

_____________________________________

Q48 - Severe brain injury with multiple skull fractures in ER. What should you do?

A - Secure airway
B - Don’t move the spine
C - CT spine
D - Call neurosurgery

Answer: A
All will be needed “at some point”.
But priority goes to airway.

_____________________________________

19
Q49 - Young male, RTA, vitally stable, unable to move arms and legs.
What is the best way of intubation?

A - Orotracheal
B - Nasotracheal
C - Cricothyrotomy
D - Tracheostomy

Answer:
Nothing in the scenario indicates the need for airway management. Inability to move an arm or
leg doesn’t constitute an indication for intubation.

The question wants to test your ability to recognize the need to minimize neck mobility during
intubation.

During airway management, patient cervical spine should be maintained in a neutral position
with the rigid cervical collar removed and manual in-line stabilization (MILS) applied
MILS should be applied by another provider with hands on the sides of the patient’s head but not
impairing mouth opening.
All airway maneuvers such as BVM, oral airway placement, cricoid or external laryngeal
manipulation, and intubation with either DL or VL will cause some degree of cervical spine
movement. This is usually in the order of 2-4mm.
Care should be taken to maintain neutral spine positioning and not place the patient in the classic
“sniffing” position with full extension of the atlanto-axial joints and flexion of the lower cervical
vertebrae.
Treat all airway management of patients with suspected spine injury as difficult and prepare
accordingly.
Rapid Sequence Intubation and your airway device (Direct or Video Laryngoscopy) of choice is
often safe.
Routine use of the bougie is recommended.

_____________________________________

Q50 - Patient MVA with pelvic fracture managed with pelvic binder.
What fluid you will give?

A - Albumin
B - LR
C - Hypertonic saline
D - Dextrose

Answer: B
Crystalloid resuscitation, followed by early blood transfusion according to the patient’s response.

_____________________________________

20
Q51 - Patient with RTA. He is stable and conscious with a patent airway. His blood pressure is
105/90 mmHg. His GCS score is 15.
What is the most appropriate next step?

A. IV Fluid
B. FAST

Answer: A
IV fluid administration is part of C in the primary survey, while FAST is an adjunct to primary
survey.

_____________________________________

21
Acute Care
Q1 - Patient known to have liver cirrhosis with ascitis, presents with umblical hernia that is
reducible and uncomfortable.
What is the most appropriate management?

A - Do operation immediatly
B - Do paracentesis and after that direct do operation
C - Don’t do operation now

Answer: C
Umbilical hernia in the presence of liver cirrhosis is a common clinical dilemma faced by
surgeons. Umbilical hernia is in large part due to the increased intra-abdominal pressure in the
presence of ascites, which leads to development of hernia through the umbilical fascial defect.
Commonly accepted indications for umbilical hernia repair are symptoms and development of
secondary complications. One crucial variable in deciding for or against recommending
umbilical hernia repair is surgical risk, which is known to be significantly higher in patients with
evidence of portal hypertension.

The high mortality rate may explain the general tendency among surgeons to avoid elective
umbilical hernia repair in patients with signs of portal hypertension. Emergency umbilical hernia
repair is associated with a higher morbidity rate than elective surgery. Specifically, failure to
wean ventilation, postoperative sepsis, and return to the operating room were more common in
the emergency setting for the patients with ascites and/or esophageal varices.

For the patients with ascites and/or esophageal varices, predictors of postoperative mortalities are
MELD score higher than 15, age older than 65 years, albumin level less than 3.0 g/dL, and
preoperative sepsis.

Optimizing the patients with liver cirrhosis before elective umbilical hernia repair is crucial to
minimizing postoperative complications and reducing recurrence. Such optimization includes
low salt intake, free water restriction, and use of diuretics, such as furosemide and
spironolactone. Large-volume paracentesis and intravenous infusion of salt-poor albumin can
help to control ascites. Another important consideration in treating patients with liver cirrhosis
who develop ascites and present with umbilical hernia is to assess the patient for suitability for
liver transplantation because new onset of ascites in patients with liver cirrhosis portends poor
prognosis with a 50% mortality at 2 years. If the patient is a candidate for liver transplantation,
umbilical hernia can be repaired at the time of transplantation surgery.

Risk of recurrence of umbilical hernia can be reduced by maximizing medical treatment of


ascites. If ascites is refractory to medical management, several options are available, such as
large-volume paracentesis and albumin replacement, the transjugular intrahepatic portosystemic
shunt (TIPS) procedure, and temporary placement of peritoneal dialysis and peritoneovenous
shunt. The TIPS procedure has been shown to successfully control ascites in 80% to 90% of

22
patients who are refractory to medical management, and it has been shown to be superior to
large-volume paracentesis in terms of survival.
Use of synthetic mesh has been shown to reduce the recurrence rate in patients without cirrhosis
undergoing umbilical hernia repair. However, in patients with cirrhosis, there is a concern for
impaired mesh ingrowth and wound infection in the presence of ascites. Therefore, one needs to
balance risk of infection and risk of recurrence in deciding whether to use mesh in patients with
liver cirrhosis who present with umbilical hernia.

In conclusion, umbilical hernia repair in the presence of signs of portal hypertension, such as
esophageal varices and ascites, is associated with significant morbidity and mortality.
Emergency surgery for umbilical hernia repair is more commonly required for this group of
patients than for the general population. Compared with elective surgery, emergency umbilical
hernia repair is associated with higher morbidity rates. However, even elective surgery is
associated with significant risk of adverse outcome in this high-risk group. Therefore, careful
selection of surgical candidates is crucial. Elective repair of an umbilical hernia should be
avoided in patients with predictors of postoperative mortality, such as age older than 65 years,
MELD score higher than 15, and albumin level less than 3.0 mg/dL.

_____________________________________

Q2 - Post colonic surgery, a patient develops abdominal pain and distention. He is vitally stable.
There is fresh blood and coffee ground aspirate from the NG tube.
What is the diagnostic modality of choice?

A - Upper GI endoscopy
B - Abdominal CT

Answer: A
The only relation between recent colonic surgery and upper GI bleeding is development of stress
ulcer. This would account for upper GI bleeding but has no relation to distension.

_____________________________________

23
Q3 - 27-year-old obese woman presents with right iliac fossa pain associated with anorexia,
nausea, and vomiting. On examination, there is moderate right lilac fossa tenderness. Labs:
leukocytosis
What is the most appropriate management?

A - Open surgery
B - CT
C - US
D - Diagnostic laparoscopy

Answer: D
Although imaging is always an option when it comes to evaluating acute abdominal pain, it is
not expected to change management of this patient. This lady is tender and needs to be explored.
The choice between open or laparoscopic approach depends on several factors, among which,
uncertain diagnosis and morbid obesity are the most solid indications to favor laparoscopy.

_____________________________________

Q4 - A patient with history of appendectomy 2 years ago comes with picture of bowel
obstruction. He has stable vitals. Abdominal exam shows minimal diffuse tenderness. Bowel
sounds are exaggerated. WBC is11.5. Xray shows multiple fluid levels.
What is next step of management?

A - Echo
B - Ultrasound
C - CT abdomen
D - Laparotomy

Answer: C
Adhesive bowel obstruction, for conservative treatment.
Certainly, surgery is not your first line of management. It is reserved for patients who present
with - or develop- complications of bowel obstruction (ischemia or perforation) or for those
patients who fail conservative treatment. Laparoscopy is preferred over laparotomy as
laparotomy will incite more adhesions.

Among the options provided, US and ECHO have no role in evaluating bowel obstruction.
Laparotomy, certainly not.
We are left with CT, which can be done though it doesn’t change management, since diagnosis is
already made clinically and radiologically.
Majority of patients with adhesive small bowel obstruction respond to conservative treatment.

_____________________________________

24
Q5 - A young female with chest pain that started after she arrived by flight journey.
What is the management?

A - Thoracostomy tube
B - Thoracotomy
C - Observation

Answer: A
Spontaneous pneumothorax
Should ideally be confirmed by findings in clinical examination and by a chest X-ray. If none
given in the question, go for chest tube insertion.

_____________________________________

Q6 - Female patient presents with red tender swelling in the axilla with history of repeated black
headend large pore skin in the same area.
What is the treatment?

A - Immediate surgery
B - Topical antibiotic
C - Cold compressor
D - Oral antibiotic

Answer: A
Hidradenitis suppuritiva
For cases of HS, treatment is prolonged courses of oral antibiotics. If abscess develops, incision
and drainage.
For repeated episodes and chronically scarred axillary skin, consider excision of the affected area
and skin grafting it.
_____________________________________

Q7 - Patient pulled out from fire has hoarseness of voice, carbonaceous sputum. Auscultation
reveals bilateral crackles.
What is the most likely diagnosis?

A - Inhalation injury.
B - Carbon monoxide poisoning.

Answer: A
This is a picture of inhalation injury. Carbon monoxide poisoning, in the other hand, typically
occurs from breathing in carbon monoxide at excessive levels. Symptoms are often described as
"flu-like" and commonly include headache, dizziness, weakness, vomiting, chest pain, and
confusion. Large exposures can result in loss of consciousness, arrhythmias, seizures, or death.
_____________________________________

25
Q8 - Patient with abdominal pain. Imaging shows: increased thickness of sigmoid colon with
thumbprint sign.
What is the next step?

A - Colonoscopy
B - Diagnostic laparoscopy
C - Exploratory laparotomy

Answer: A
Thumbprinting is a sign of infectious colitis. It is caused by edema of colonic wall.

_____________________________________

Q9 - 60-year-old man known to have heart disease on Aspirin and Plavix presents with severe
epigastric pain radiating to the back. Examination revealed generalized tenderness and absent
bowel sounds. Chest Xray showed air under diaphragm.
What is the most likely diagnosis?

A - Perforated peptic ulcer


B - Acute pancreatitis
C - Gastric outlet obstruction
D - Gallstone disease

Answer: A
This is a straightforward question. Pneumoperitoneum (in absence of history of recent surgery or
drains) is an evidence of hollow viscus perforation, perforated DU in this case.

_____________________________________

Q10 - Patient presents with epigastric pain radiating to the back with tenderness on abdominal
exam. On rectal exam, there is an empty collapsed rectum. X-ray shows air under the diaphragm.
What is the diagnosis?

A - Pancreatitis
B - Duodenal perforation
C - Pneumothorax

Answer: B
Perforated DU.

_____________________________________

26
Q11 - Case of appendicular abscess 10 x 15 cm
what is the management?

A - Open drainage
B - Percutaneous drainage
C - Laparoscopic appendectomy with drainage
D - Open appendectomy with drainage

Answer: B
Admission, IVF, bowel rest, IV antibiotics, and image-guided percutaneous drainage.
_____________________________________

Q12 - 60-year-old patient with chronic liver disease and ascites comes with umbilical hernia.
How would you manage it?

A - Laparoscopic hernial repair


B - Repair with waterproof mesh
C - Wait till ascites is treated first
D - Do paracentesis and repair

Answer: C
Occurrence of PUH in cirrhotic patients with ascites is a very common clinical scenario.
Attempts to repair these hernias in presence of uncontrolled ascites is associated with very high
failure rates.
Once surgical indication for repair is posed, a close coordination between the treating
hepatologist and the surgeon is very important. Strategies to control ascites should be
implemented. Surgery is done once ascites comes under a reasonable control.
If the patient presents emergently (with obstructed or strangulated hernia or with leaking ascites
through the typically thinned out overlying skin), emergency repair should be carried out along
with drain placement for control of ascites.
_____________________________________

Q13 - 33-year-old male 5 days after appendectomy comes with fever and discharge from the
wound.
Which of the following would be the best step in management?

A - CT abdomen
B - Antibiotics

Answer:
Wound infection
Should be managed by opening the skin sutures, drainage of pus and cleaning the wound.
Antibiotics are certainly needed.
CT is not routinely done for simple wound infections (in absence of other indicators of
intraabdominal or pelvic collections.
_____________________________________

27
Q14 - An immune compromised patient comes with perianal swelling, discharge, and pain. On
examination, he has crepitus.
What is the best management?

A - Debridement
B - Aspiration
C - Antibiotics

Answer: A
Fournier gangrene is a form of necrotizing fasciitis that develops in perineal and/or genital area.

Impaired immunity (e.g., due to diabetes) is known to increase susceptibility to Fournier


gangrene.

Early, aggressive intervention is critical, as the condition is associated with a high mortality rate.
Surgery is necessary for definitive diagnosis and excision of necrotic tissue. Along with
debridement, surgical procedures may include suprapubic tube placement, and fecal diversion.

_____________________________________

Q15 - 24-year-old male, medically and surgically free, presents with features of small bowel
obstruction.
What is the most common cause?

A - Adhesions
B - Meckel’s diverticulum
C - Malignancy
D - Hernia

Answer: D
The commonest cause of small bowel obstruction in people with previous abdominal surgeries, is
adhesions.

In patients with virgin abdomen, it is hernia.

_____________________________________

28
Q16 - Patient with perforated duodenal ulcer.
What is the proper surgical procedure?

A - Repair ulcer with omental patching


B - Vagotomy

Other recall:
Q - Anterior duodenal perforation management?

A - Omental patch

Answer: A
Peritoneal washout and Graham’s patch.
_____________________________________

Q17 - 50-year-old patient with periumbilical pain and vomiting. On examination, there is
abdominal distention with exaggerated bowel sounds. Labs show high amylase and ABC count.
What is the most likely diagnosis?

A - Pancreatitis
B - Small bowel obstruction
C - Mesenteric occlusive ischemia

Answer: B
Central abdominal pain, abdominal distention and increased bowel sounds are all features of
small bowel obstruction.
Now where does high amylase fit in this scenario?!! Remember that there are many causes of
hyperamylasemia (other than acute pancreatitis). Bowel obstruction is one of them.
_____________________________________

Q18 - 56-year-old man, atrial fibrillation on warfarin, develops sudden right lower quadrant pain
after lifting heavy object. On examination, he has a tender mass in RLQ with negative cough
impulse.
What is the most appropriate management?

A - Angioembolization
B - Rest and analgesic
C - Reassure
D - Stop anticoagulant
Another recall, reverse anticoagulant

Answer:
Management of rectus sheath hematoma is protocolized and progresses from rest and analgesia
all the way to blood transfusion, reversal of anticoagulation and angioembolization
The choice of treatment depends on certain factors (that are not all available in this scenario).
_____________________________________

29
30
31
Q19 - Elderly patient presents with acute appendicitis, managed conservatively, and discharged 7
days later. On imaging there is appendiceal mass with no collection.
What is the best management?

A - No further intervention
B - Open appendectomy after 12 W
C - Lap appendectomy after 12 weeks

Answer:
Acute appendicitis is one of the most frequent acute surgical pathologies. The inflammation in
acute appendicitis may sometimes be fixed by the patient's own defense mechanisms, by the
formation of an inflammatory mass (an appendiceal phlegmon) or a circumscribed abscess (an
appendiceal abscess), often presenting as a palpable mass days following the onset of symptoms.
This complication occurs in 2 to 7% of all cases of appendicitis.

There are 3 methods for treatment of appendiceal mass: emergency surgery, conservative
management followed by interval surgery, and totally conservative management without interval
surgery. The surgeon must consider clinical symptoms and results of investigations in each
particular case when choosing an appropriate treatment method.

The most widespread method of treatment is considered the nonoperative method by Ochsner
(1901). This method implicates starting treatment with broad-spectrum antibiotics, IV fluid
hydration and bowel rest. In case of improvement in the patient's condition, interval surgery is
indicated after 8 to 12 weeks. In case of existence or formation of appendiceal abscess, US or
computed tomography-guided percutaneous drainage is indicated. If the patient's condition is not
improved, surgical intervention must be performed.

Immediate appendectomy may be technically demanding because of the distorted anatomy and
difficulties in closing the appendiceal stump due to the inflamed tissues. In fact, many of the
attempted “immediate appendectomies” will end up with some form of colonic resections
(ileocecectomy or right hemicolectomy). So, this approach should not be adopted except in
patients who initially present with generalized peritonitis or marked septic manifestations that
make them poor candidates for non-operative approach.

Conservative management with interval appendectomy has traditionally remained the gold
standard management. The need for interval appendectomy after a successful nonsurgical
treatment has recently been questioned as the risk of recurrence is relatively small. This led
“routine” interval appendectomy to fall out of favor and it is now reserved for a certain subset of
patients where an underlying pathology needs to be ruled out. Patients over the age of 40 with
successful conservative treatment of appendicular mass should undergo a diagnostic colonoscopy
4 - 6 weeks after resolution of the acute attack, and a serious consideration to interval
appendectomy should be given several weeks later.
_____________________________________

32
Q20 - Young woman comes to ER with unilateral limb edema. She has been using OCP
What is the best diagnostic modality?

A - Duplex US
B - CT scan
C - MRI

Answer: A
OCPs increase the risk of development of DVT and other thromboembolic events.
_____________________________________

Q21 - Obese patient comes with right iliac fossa pain and tenderness.
What is the best imaging modality to diagnose appendicitis?

A - CT abdomen
B - Ultrasound abdomen

Answer: A

US performs poorly in obese patients.

_____________________________________

Q22 - Patient with atrial fibrillation presents to ER with severe abdominal pain and all signs and
symptoms of mesenteric ischemia. He is unstable.
What is the best management?

A - Colonoscopy
B - Diagnostic laparoscopy

NB. exploratory laparotomy not an option

Answer:
Such a patient with acute abdomen and a picture of bowel ischemia should be explored.
Laparoscopy is very ill-advised in hypotensive patients as the pressure of pneumoperitoneum can
compromise venous return, hence reducing cardiac preload and subsequently, afterload, further
compounding the pre-existing hypotension.
I would go for laparotomy. It is the only right decision here.

_____________________________________

33
Q23 - Pregnant female presents with right abdominal pain (lower or upper) associated with
nausea and vomiting.
Ectopic pregnancy was ruled out.
What is the most likely diagnosis?

A - Cholecystitis
B - Appendicitis
These are the only options I remember

Answer:
Upper, cholecystitis.
Lower, appendicitis.

_____________________________________

Q24 - A pregnant lady with acute cholecystitis


How to manage?

Answer:
Acute cholecystitis in pregnant women is treated no differently than that in non-pregnant. The
recommendation is to go for cholecystectomy regardless of gestational age.
Limiting surgery to second trimester is obsolete.

Traditionally, the recommendation for non-emergent procedures during pregnancy has been to
avoid surgery during the first and third trimesters to minimize the risk of spontaneous abortion
and preterm labor, respectively. This has led some authors to suggest delaying surgery until the
second trimester and that the gestational age limit for successful completion of laparoscopic
surgery during pregnancy should be 26 to 28 weeks. These recommendations are not supported
by good quality evidence; recent literature has demonstrated that pregnant patients may undergo
laparoscopic surgery safely during any trimester without an increased risk to the mother or fetus.
Both laparoscopic cholecystectomy and appendectomy have been successfully performed late in
the third trimester without increasing the risk of preterm labor or fetal demise. Importantly,
postponing necessary operations until after parturition has been shown in some cases, to increase
the rates of complications for both mother and fetus.

_____________________________________

34
Q25 - Patient underwent splenectomy, pancreatectomy and liver laceration after gunshot comes
to ER with generalized tenderness.
What is the next step in management?

A - Exploratory Laparotomy
B - Images

Answer:
This patient has multiple potential reasons for his presentation.

Management depends on time frame of presentation and systemic manifestations and presence or
absence of signs of peritoneal irritation (that includes, but not limited to tenderness)

Causes include bleeding, hematoma, infected hematoma, anastomotic leak, collection, missed
bowel injury, among others.

In absence of hemodynamic instability and / or peritonitis, imaging by CT is appropriate.


Compromised hemodynamics and / or peritonitis is indication for exploration.

_____________________________________

Q26 - 46-year-old male presents to ER with RUQ pain, fever, chills and rigors and SOB. He is
three weeks post laparotomy for perforated duodenal ulcer. On examination, he is febrile and
tender in the right hypochondrium.
Chest X-Ray showed right lower lobe atelectasis with some pleural effusion.
What is the most appropriate next step?

A - CT chest
B - Abdominal Ultrasound
C - Exploratory laparotomy

Answer: B
Everything points towards post-operative collection in the upper abdomen (sub-hepatic or more
likely sub-diaphragmatic)
Changes in the chest are reactive.
These collections rarely need surgical drainage
Chest CT is not expected to show more than what we have already seen In X-ray.
Ideally, one would go for abdominal CT with both IV and on table oral contrast to image the
collection and to rule out a persistent leak from the duodenum that has caused the collection to
develop. This option is not given. So go for the next best thing, which is US.
US will show the collection, and that should be treated with percutaneous US- guided drainage.
Other aspects of management include good hydration and systemic antibiotics.

_____________________________________

35
Q27 - Patient sustains gunshot wound to the thigh. He has paresthesia and can’t move his limb.
Examination showed weak pulses in the affected limb.
What is the best management?

A - Exploration in OR
B - Exploration now
C - Angiography

Doctor this seems to be Compressive hematoma, shouldn’t we do bedside evacuation (same as


hematoma post thyroid surgery)

Answer: A
Expanding hematoma is a hard sign of vascular injury.
Paralysis and paresthesia can be caused by compartment syndrome or by direct nerve injury from
the bullet.

This patient should be explored in OR.

Unlike neck hematomas developing after thyroidectomy which are typically managed first by
bedside evacuation (to address the airway compression) then patient should be taken to OR for
achieving hemostasis, this thigh hematoma should be explored in OR with proper exposure and
securing proximal and distal control of the affected vessel. The affected nerve should be
explored and examined for possible injury.

_____________________________________

Q28 - Scenario with features of bowel obstruction (nausea, vomiting, abdominal distension, and
central abdominal pain). Examination shows abdominal distension with diffuse tenderness and
exaggerated bowel sounds.

What is the next best initial step?

A - Barium enema
B - Colonoscopy
C - Examine the groin
D - Laparotomy

Answer: C
Examine the groin for the possibility of irreducible/obstructed groin hernia.

_____________________________________

36
Q29 - 56-year-old patient presents with left lower quadrant pain and change in bowel motion.
Upon examination, patient has fever and left lower quadrant tenderness. Imaging showed
thickened bowel.
What is the diagnosis?

A - Diverticulitis
B - Sigmoid mass
C - Ruptured ovarian cyst
D - Pyelonephritis

Answer: A
Very common scenario in SMLE.
_____________________________________

Q30 - 44-year-old man presents with perianal pain. It is accompanied by loose stools which are
sometimes bloody. He also complains of colicky abdominal pain on and off for a few months.
On examination, there is a hot, tender perianal mass with fluctuation. Vitals: BP: 123/65 HR: 106
T: 39.1 O2: 100%. Labs: WBC 19.
What is the most appropriate next step of management?

A - IV antibiotics
B - I&D
C - MRI pelvis
D - Abdominal ultrasound

Answer: B
Although IBD is likely here and the patient should be investigated for that possibility, the acute
management will still be the same.
A perianal abscess would still need to be drained.

_____________________________________

37
Q31 - Appendiceal mass completely recovered on only antibiotics.
What is the next appropriate step in management?

A - Lap appendectomy on 6 weeks


B - Open appendectomy on 3 – 6 months
C - No further evaluation

Answer: Age is very important.

Routine interval appendectomy after successful conservative treatment of acute appendicitis is


no longer indicated as the risk of recurrent attack is very low.
Exceptions are those with free fecolith or older patients where possibility of underlying
malignancy can’t be reliably ruled out

Here you find a nice overview of the topic


https://www.ncbi.nlm.nih.gov/pmc/articles/PMC3351492/pdf/WJGS-4-83.pdf

_____________________________________

Q32 - 32-year-old female, 16 days post CS, presenting with fever not responding to antipyretic
and antibiotics. She has previous multiple CS with adhesions. CT showed small bowl fistula.
What is your initial management?

A - NPO and start pancreatic and gastric secretion inhibitors


B - Resect and anastomose
C - Restrict to soft diet
D - Colostomy

Answer:
Fistula developing in 16 days is a very unlikely happening as fibrous fistula tract normally needs
4-6 weeks to form.

The INITIAL step is certainly NPO.


Starting Octreotide and other measures to decrease pancreatic and/or gastric secretion is not
routinely needed. You need to know the location of the fistula along the length of small bowel
(proximal or distal) and its daily output. Distal fistulas with low output don’t need such
approaches

Decision for resection can’t be considered as initial management.


Colostomy is not needed in small bowel surgery.

_____________________________________

38
Q33 - 32-year-old female, 16 days post CS, presenting with fever not responding to antipyretic
and antibiotics. She has previous multiple CS with adhesions. CT showed small bowl fistula.
What is the most appropriate management?

A - NPO and start pancreatic and gastric secretion inhibitors


B - Resect and anastomose
C - Restrict to soft diet
D - Colostomy

Answer:
This scenario is that of small bowel injury rather than a fistula

Difficult C-section by virtue of adhesions caused by previous C-sections can be complicated by


small bowel injury. If recognized immediately, management is easy with primary repair of small
enterotomies and resection/ anastomosis of the bigger ones.

If missed, small bowel injuries present days after surgery with sepsis and peritonitis. Some
“contained” injuries continue to leak gradually- but surely- till they present as Enterocutaneous
Fistula. This typically takes anywhere from 4 to 6 weeks as this is the time needed for fibrous
tissue formation.

Now back to the scenario. We got to admit that 16 days are too long for someone to present with
missed small bowel injury and too soon to present with a fistula.

NPO and suppression of gastric and pancreatic secretions would be an option for proximal, high
output fistulas. Nothing in the question points to that.

Diet manipulation would be a reasonable approach to distal, low output fistulas. Again, nothing
in the question points to that end either.

Colostomy has no role in management of small bowel pathologies.

Resection/ anastomosis would be the way to go.

_____________________________________

39
Q34 - 24-year-old male presents with RLQ pain and fever, diagnosed with acute appendicitis and
was taken to OR for appendectomy using Gridiron incision. During surgery, the surgeon found a
normal appendix.
What is the appropriate action?

A - Close the patient


B - Resect the appendix
C - Look for Meckel's diverticulum
D - Transfer to laparotomy

Answer: C
Normal appendix at exploration should prompt the search for another pathology that can explain
patient’s symptoms.
Meckel's diverticulum, Terminal ileitis, ovarian pathologies - in females - are all potential
sources of the pain and should be actively sought for.

_____________________________________

Q35 - 78-year-old male after contaminated abdominal surgery, admitted to ICU, and received 6
L of crystalloid IV fluid and 1 PRBC in the last 12 hours. Broad spectrum antibiotics were
administered.
Vitals: BP 70/45, HR: 125, T: 38.6, O2: 93%
Labs: WBC 8
What is the most appropriate IV intervention?

A - Albumin
B - 2 L crystalloid fluid
C - Norepinephrine
D - Furosemide

Answer: C
Septic shock. After fluid resuscitation of 6 L next step is EARLY inotropic support as per
surviving sepsis campaign.

_____________________________________

40
Q36 - Post appendectomy day 5 (presents on day 8) with right iliac fossa pain. Purulent
discharge was noted. BP stable, temp high.
What is the most appropriate next step?

A – IV antibiotic
B - Percutaneous drainage
C - Exploratory laparotomy

Answer:
I suppose you are talking about a patient post appendectomy coming back with fever and
purulent discharge from the wound. This is a superficial surgical site infection (SSI). You need
imaging to rule out intra-abdominal or pelvic collections (deep SSI).

Superficial SSIs are treated with local wound care and systemic antibiotics while deeper ones
require percutaneous image-guided drainage as well as systemic IV antibiotics
_____________________________________

Q37 - Pathophysiology regarding patient with appendicitis complicated by appendicular mass


(Abscess), patient was feverish (38.7)?

A - Peripheral Vasoconstriction
B - Decreased Cardiac Index
C - Redistribution of Blood
D - Bradycardia

Answer: C
Distributive shock.
_____________________________________

Q38 - Patient comes with acute appendicitis and 10 x 12 cm abscess and a free fecalith.
What is the most appropriate management?

A - Open appendectomy with drainage


B - Laparoscopic appendectomy with drainage
C - Drainage only.

Answer: B
Standard treatment of appendicular abscess is percutaneous drainage. Presence of a free fecolith
makes this option problematic as this fecolith – if left in place - will serve as a nidus of sepsis
that self-perpetuates.

Best approach will be laparoscopic appendectomy with drainage of the pus collection and
removal of the fecolith.
_____________________________________

41
Q39 - Patient underwent open appendectomy 2 weeks ago for perforated appendicitis, presents
with feculent discharge from the wound. Abdomen is rigid and the patient is vitally stable.
What is the best next step?

A - CT abdomen
B - Laparotomy
C - Ultrasound

Answer: B
Appendicular stump leak.
Imaging is redundant.
Patient should be surgically explored, preferably laparoscopically (if that comes among the
options) or alternatively through a lower midline laparotomy.

_____________________________________

Q40 - Patient known case of cardiomyopathy and atrial fibrillation presents with signs and
symptoms of acute mesenteric ischemia. Abdomen is rigid.
What is the best next step?

A - Heparinization
B - Diagnostic Laparoscopy.

Answer: B
Presence of peritonitis in this context is an indication for surgical exploration.
Diagnostic laparoscopy would be a very reasonable approach.

_____________________________________

Q41 - A boy was playing football and felt sudden pain in his groin while kicking the ball or
aiming to the goal. There is a swelling at the groin. It is irreducible and there is no cough
impulse. Vitals showed fever and WBCs were high.
What is your management?

A - Aspiration to rule out hematoma


B - Give analgesics and check after 8 hours
C - Surgical exploration for hernia
D - Forced reduction manually

Answer: C
Looks like an irreducible inguinal hernia. Emergency exploration and repair is the way to go.

_____________________________________

42
Q42 - 24-year-old male comes with RLQ pain, positive rebound tenderness after 2 hours of
playing football. His WBC count 6 (normal), INR 1.1 (normal).
Which of the following is the most appropriate next investigation?

A - CT abdomen
B - No need for further investigation, and follow up after 1 week
C - US or Xray (not really sure which one was there)

Answer:
For a 24-year-old male with right lower quadrant pain and positive rebound tenderness, top
differential would be acute appendicitis. Normal white count doesn’t rule it out.

I can’t understand why do we have to imagine rectus sheath hematoma just because the guy was
playing football!!

_____________________________________

Q43 - What is the best way to identify appendix in surgery?

A - From internal iliac artery


B - Palpate ileocecal valve
C - Convergence of taeniae coli
D - Ileocecal valve

Answer: C
Convergence of the three taeniae coli will lead to the base of the appendix.

_____________________________________

Q44 - 17-year-old male, RIF pain and nausea. He has positive rebound tenderness and obturator
sign. Lab showed leukocytosis. (High Alvarado score).
Asking what next?

A - Appendectomy
B - CT
C - US

Answer: A
Acute appendicitis is a clinical diagnosis.
In face of a very suggestive clinical presentation like the one in hand, radiological studies
become redundant.

_____________________________________

43
Q45 - 35-year-old patient comes with intestinal obstruction. He has negative surgical and
medical history. He is stable.
What is the most common cause of his obstruction?

A - Hernia
B - Adhesion
C - Malignancy
D - Meckel’s diverticulum

Answer: A
Most common cause of small bowel obstruction in virgin abdomen is hernia. In patients who had
previous abdominal surgeries, it is adhesions.

_____________________________________

Q46 - Patient with atrial fibrillation comes to ER with severe abdominal pain and all signs and
symptoms of mesenteric ischemia. He is unstable.
What is the most appropriate management?

A - Colonoscopy
B - Diagnostic laparoscopy
C - Don’t remember
NB. exploratory laparotomy not an option

Answer:
Mesenteric ischemia, acute abdomen, hemodynamic instability…
There must be exploratory laparotomy in the options.

_____________________________________

Q47 - 24-year-old male presents to ER and diagnosed as acute appendicitis and was taken to OR,
they did “mention the name of a weird incision” and then found the appendix normal.
What to do next?

A - Check for Meckel’s diverticulum


B - Remove the appendix and close the abdomen

Answer: A
The weird incision is McBurney’s
The patient was taken to OR because he had convincing clinical findings to justify exploration. If
appendix is found to be normal, search for an alternate pathology that can account for patient’s
presentation.
_____________________________________

44
Q48 - Peritonitis. X-ray shows, dilated colon >10 cm. CT, no mechanical obstruction.
How to manage megacolon?

A - Total colectomy
B - Partial colectomy with stoma
C - Decompressive rectal tube

Answer:
If this question comes without “peritonitis”, you go for C

But here there is peritonitis, possibilities are perforated colon where you will need to do
segmental resection of that part, or “Toxic Megacolon” where you will need to go for total
colectomy
There must be more data in the question. Get a better recall. If you are sure this is a correct
recall, go for total colectomy
In a more popular recall, there is no peritonitis. So, it is Ogilvie’s Syndrome we are dealing with,
and the answer would be sigmoidoscopy and placement of rectal tube. Please note that some
recalls mentioned colonoscopy instead of sigmoidoscopy, it is basically the same thing in this
context.

_____________________________________

Q49 - Patient with intestinal obstruction and perforated and the doctor decided to do EX LAP
what is the contraindications?

A - Nitrous oxide
B - Propofol

Answer: A
The question is probably about laparoscopy, not laparotomy
NO is contraindicated in laparoscopy as it causes bowel distension and compromises exposure.

_____________________________________

Q50 - Alderley patient comes to ER with severe headache then suddenly deteriorates with
decreased level of consciousness. CT showed hematoma.
What is the best next step?

A - Admit to ICU
B - Give mannitol
C - Evacuate in ER

Answer: B
_____________________________________

45
Vascular Surgery
Q1 - 40-year-old female with 12-year history of varicosities, complains of lower limb edema and
heaviness when standing. Distal pulses are intact and there are bilateral varices in great
saphenous vein territory.
What is the management?

A - Sclerotherapy
B - Endovascular laser
C - Thermal ablation

Answer: C

_____________________________________

Q2 - Old patient long history of leg claudication, presents with leg pain and ABI <0.3.
What is the appropriate management?

A - Amputation
B - Thrombolysis
C - Thrombectomy

Answer: A
Thrombectomy and / or thrombolysis is not an option for this “long history”. If the question
gives you options like angioplasty or bypass that would be a very reasonable approach. These
options apparently are not given, however. This leaves you with one option, amputation

_____________________________________

Q3 - 27-year-old male presents with left leg pain that started 4 days ago. Examination showed
left leg edema (I believe they mentioned “suspicious of DVT” not sure). He also had chest pain
and dyspnea that started 2 days ago.
How will you manage?

A - Aspirin
B - Warfarin
C - Thrombolysis
D - Unfractionated heparin

Answer: D

_____________________________________

46
Thoracic Surgery
Q1 - A healthy patient presenting with dysphagia. No family history of esophageal cancer.
What is the strongest risk factor for esophageal cancer?

A - Esophageal stricture.
B - Barrett’s esophagus
C - GERD
D - Smoking

Answer:
GERD —-> adenocarcinoma
Smoking—-> squamous

_____________________________________

Q2 - Lung nodule measures 8mm, asymptomatic


What to do?

A - Follow up
B - CT scan
C - Biopsy

Answer:
If that was diagnosed by CT … then Biopsy
If not… then CT first

_____________________________________

47
Q3 - Patient with weight loss and dysphagia diagnosed by EGD and biospy as a case of
adenocarcinoma.
How to know T stage of cancer?

A - Abdominal Us
B - CT chest and abdomen pelvis
C - Endoscopic US

Answer: C
Although CT is needed to evaluate metastatic lesions, it can’t reliably determine the depth of
penetration.
_____________________________________

Q4 - SCC of lung for surgical resection. No metastasis.


What to do before surgery?

A - Chemo
B - Repeat pet scan after 3 months
C - Radiation
D - PFT

Answer: D
_____________________________________

Q5 - Patient has 10.5 cm lung nodule.


What to do next?

A - Follow up with CT scan in 6 months


B - Needle biopsy

Answer: B
_____________________________________

Q6 - Asthmatic patient complaining of shortness of breath in supine position. He also reported


dysphagia. CT, midline mass.
What is the most likely diagnosis?

A - Lymphoma
B - Thymoma
C - Goiter

Answer: C, The question doesn’t tell you if the “midline” mass is in the neck or in the
mediastinum. I would go for C, however.
_____________________________________

48
Two related recalls:

Q7 - Patient with pneumonia on medication presents with parapneumonic effusion. The pleural
pH is 7.1
What is your next action?

A - Continue medical treatment


B - Chest tube drainage with antibiotics
C - Small pigtail drain with antibiotics

Q8 - Patient treated with multiple antibiotics for pneumonia. On day 5 he developed fever with
dullness upon percussion. Imaging revealed right pleural effusion and labs tell it is exudative.
What to do?

A - Ceftriaxone and Azithromycin


B - Chest tube and another antibiotic
C - Small pleural drain with piperacillin-tazobactam

Answer: B
This question wants to test your knowledge about management of transudate vs. exudate

Although both, pigtails and chest tubes are used to drain pleural effusions in general. In this
scenario, however, they want you to conclude that the effusion is pus (empyema) based on its
low PH, for which a chest tube drainage is the right answer.
In everyday practice, most cases of pleural effusion can be effectively managed by pigtail pleural
drain. In fact, pigtails are preferred over chest tubes because they are better tolerated by patients
due to the minimal pain and discomfort they cause.
But this is SMLE. They want you to pick the clues: the question even mentioned “small” pigtail
pleural drain for you to conclude that it may not be able to drain the thick pus pf empyema.

_____________________________________

49
Gallbladder Diseases

Q1 - A patient comes with RUQ pain. Ultrasound findings: Multiple gallstones with thickened
gallbladder wall, pericholecystic fluids.
What is the best intervention?

A - Percutaneous drainage
B - Laparoscopic cholecystectomy

Answer: B
Acute cholecystitis.
In absence of contraindications to surgery or to GA (e.g., poor overall health), all cases of acute
cholecystitis should be offered laparoscopic cholecystectomy.

_____________________________________

Q2 - Pregnant has RUQ pain. US showed cholelithiasis.


What is the best next step?

A - Lap chole
B - Open chole
C - Conservative management and reassess after pregnancy

Answer: C
Single attack of biliary colic. No signs of acute chole. Conservative for now is appropriate.

_____________________________________

Q3 - 10 weeks pregnant complaining of biliary colic for the past 5 weeks.


What is the most appropriate management?

A - Laparoscopic cholecystectomy now


B - Laparoscopic cholecystectomy after delivery
C - Laparoscopic cholecystectomy in second trimester
D - Laparoscopic cholecystectomy in third trimester

Answer: A
Recurrent biliary colic is an indication for surgery in pregnant women, for two reasons. First, it
affects quality of life, second, more frequent attacks of biliary colic heralds the development of
acute cholecystitis or other gallstone-related complications.
____________________________________

50
Q4 - 2 weeks after lap chole, patient presents with vague abdominal pain. Vitals are normal
including temperature. CT shows large collection in the subhepatic area.
What is the most appropriate next step?

A - ERCP with stenting


B - Laparoscopic drainage
C - CT guided drainage
D - Operative drainage

Answer: C
Image-guided drainage by interventional radiology .

_____________________________________

Q5 - A patient is presented with jaundice. US shows a shrunken gallbladder with dilatation of


intrahepatic ducts.
What is the most likely diagnosis?

A - Acute Cholecystitis
B - Gallbladder Stone
C - CBD Stone
D - klatskin tumor

Answer: C
Did US show gallstones?? Well, it should, because this scenario goes with CBD stones.

_____________________________________

Q6 - Post laparoscopic cholecystectomy presenting few days afterwards with ascites and
abdominal pain.
What will you do?

A - Open
B - Laparoscopy
C - Tapping
D - ERCP

Answer:
We will need to understand the difference between ascites and collection
This is postoperative collection. Treatment is percutaneous image-guided drainage (in this recall
they wrote it as tapping).

_____________________________________

51
Q7 - Cause of biliary colic?

A - Choledocholithiasis
B - Gallbladder stone
C - Gallbladder sludge

Answer:
Choledocholithiasis causes obstructive jaundice.
Both, gallbladder stones and sludge can cause biliary colic. If they both come in the same
question, go for gallstones.

_____________________________________

Q8 - Case of RUQ pain and tenderness and jaundice. He has High WBCs and elevated total
bilirubin (not mentioned direct or indirect). On US, multiple GB stones no pericholecystic fluid,
CBD is dilated 1cm.
(No mention of fever in the question)
What is the diagnosis?

A - Ascending cholangitis
B - Acute cholecystitis
C - Choledocholethiasis
D - Acute pancreatitis

Answer: A
Facts in the question:
- RUQ pain and tenderness
- Jaundice
- Gallstones without pericholecystic fluids.
- Dilated CBD
- High white count
- No fever

Now, it can’t be cholecystitis because US doesn’t show features of that.

Acute pancreatitis is out too.

Choledocholithiasis, explains pain, jaundice, dilated CBD. But it should have no features of
infection (tenderness and high white count)

Ascending cholangitis, in the other hand can explain all these findings (plus fever if provided in
the scenario).

_____________________________________

52
Q9 - Patient known to have gallstones presents to ER with severe RUQ pain for 6 hours. It
started after eating a fatty meal. 2 hours later the pain gradually decreased. Ultrasound shows
multiple gallbladder stones with normal wall.
What is the most appropriate management?

A - Laparoscopic cholecystectomy
B - Observation
C - Ursodeoxycholic acid

Answer: A
Persistent biliary colic.
Advise for early (not urgent) cholecystectomy as outpatient.
No indication for admission for observation
Ursodeoxycholic acid is not routinely used in management of gallstones.

_____________________________________

Q10 - Patient few days post cholecystectomy developed chest pain worse with inspiration. LUQ
pain and tenderness with dull percussion, temperature 38, basal crepitations.
What is the most likely diagnosis?

A - Sub phrenic abscess


B - Lower lobe pneumonia

Answer: A
Gallbladder is located in the right side of the abdomen, so it is probably the RUQ the question is
asking about.

Quite often, lower lobe pneumonia (whether left or right) is caused by intra-abdominal
collections below the corresponding hemidiaphragm.

Collections complicating cholecystectomy can present with right lower lobe pneumonia along
with features of the abdominal pathology.

_____________________________________

53
Q11 - 41-year-old man presents with severe abdominal pain and distension, five days after
laparoscopic cholecystectomy. Examination revealed a tender and distended abdomen.
BP 100/55
HR 103
Temp 37.9
US revealed the presence of ascites.
Which of the following is the most appropriate management?

A - ERCP
B - Percutaneous drainage
C - Exploratory laparotomy
D - Diagnostic laparoscopy

Answer: B
This can’t be ascites.
It should rather be “collection”
It should be amenable for percutaneous image-guided drainage.

_____________________________________

Q12 - During lap chole, what determines chronicity of cholecystitis?

A - Wall thickness
B - Adhesions
C - Subserosal hemorrhage

Answer: A
Adhesions can be seen in both, acute and chronic cholecystitis.
Thickened GB wall is a feature of chronic inflammation.

_____________________________________

Q13 - Patient presents with recurrent right upper abdominal pain, temperature 37.3. On lab,
elevated alkaline phosphatase, and direct bilirubin. US finding, cholelithiasis, thickening
gallbladder wall and CBD 12 mm.
What is the most appropriate management?

A - Laparoscopic cholecystectomy
B - ERCP
C - MRCP

Answer: B
Obstructive jaundice, for ERCP.
_____________________________________

54
Q14 - Patient undergoing laparoscopic cholecystectomy, as they were inserting the tubes
"laparoscopy equipment" patient develops severe bradycardia 40 bpm.
What could be the cause?

A - Cold air insufflation


B - Rapid stretch of the peritoneum

Another recall:
Q15 - Patient undergoing laparoscopic cholecystectomy. after creating pneumoperitoneum and
insertion of the trocars, the patient developed bradycardia.
What is the likely cause?

A - Cold gas insufflation


B - Rapid stretch of parietal peritoneum

Answer: B
Insertion of trocars doesn’t cause this.
It is in fact the flow of CO2 to establish pneumoperitoneum that causes sudden stretch of
peritoneum which - in turn - provokes vagal stimulation manifesting as bradycardia.
To avoid this problem, we insufflate the abdomen slowly to avoid sudden stretch of peritoneum.
Management: stop insufflation immediately, open all trocars for deflation. Some patients would
need a STAT dose of IV atropine.
HR returns to normal very quickly with these measures and the procedure can proceed smoothly
afterwards.

_____________________________________

55
Q16 - 38-year-old female presents to ER complaining of epigastric pain for 6 days, radiating to
the back, associated with multiple episodes of vomiting. She is known to have multiple small
gallstones. On examination, there is diffuse abdominal distention, epigastric tenderness, and
sluggish bowel sounds.
Labs: ALP, amylase total and direct bilirubin all within the normal range
What is the most appropriate investigation?

A - Trans abdominal ultrasound


B - Abdominal CT
C - Urine amylase
D - Abdominal fluid analysis

Answer: B
Small gallstones, epigastric pain radiating to the back and epigastric tenderness, all point towards
acute biliary pancreatitis. What is not fitting here is the fact that amylase is normal.
Now what to do to confirm diagnosis?!!

US will re-demonstrate the gallstones. Although it is important in determining the etiology of


acute pancreatitis, it doesn’t make the diagnosis of it.

Urine amylase will be detected long after serum amylase has normalized, but it won’t be still
there in 6 days.

There is nothing in the question that says there is ascites to be tapped (D)

Now let’s go to the ATLANTA criteria for diagnosis of acute pancreatitis.


You need two out of three.
- Classic clinical presentation (which we have here).
- Elevated pancreatic enzymes more than three folds upper limit (which we don’t have here).
- Radiological evidence of pancreatitis (therefore you should obtain a CT in this setting of a very
suggestive clinical presentation but normal level of pancreatic enzymes).

_____________________________________

56
Q17 - Patient had PTC for obstructive jaundice. 24 hours later, developed upper GI bleeding.
Vitals stable.
What's the appropriate initial investigation?

A - CT scan
B - Endoscopy
C - US
D - Angiography

Answer: A
Haemobilia
Best diagnostic modality is CT Angio, if bleeding spot is identified you go for conventional
Angio for embolization.

_____________________________________

Q18 - Patient after difficult prolonged ERCP, develops neck, chest, and abdominal wall surgical
emphysema.
What was injured?

A - Esophagus
B - Trachea
C - Duodenum
D - Bile duct

Answer: C

_____________________________________

Q19 - Patient presents with biliary pain and yellow sclera. He looks unwell.
Lab shows increased WBC
US shows gallstones and dilated CBD.
(No fever in the scenario)
What’s the diagnosis?

A - Ascending cholangitis
B - Choledocholithiasis

Answer: A
Some patients, especially elderly, fail to mount an adequate inflammatory reaction, hence, they
don’t show fever. In fact, hypothermia in septic shock is more worrisome than fever. Here you
have enough evidence of infection (patient being unwell and high white count).

_____________________________________

57
Q20 - Patient underwent lap chole for multiple gallstones 7 days ago, presents with vague
abdominal pain. On US there is fluid around the gallbladder (something like that) and the CBD is
9 mm.
What is the most likely diagnosis or cause?

A - CBD injury
B - Retained stone in CBD
C - Sub-hepatic collection

Answer: Incomplete scenario


Missing signs and symptoms, fever, jaundice, and abdominal tenderness?
Missing labs: WBC, Bilirubin level?!!

Subhepatic collections will present with pain, and - if infected - features of active inflammatory
process (fever, tenderness, and elevated white count).
CBD injury is either transection or side injury. If transacted, proximal ducts will be dilated. If
side injury takes place, the duct might be collapsed (unless externally compressed by a large
collection) and there will be bile collection in the gallbladder fossa and the subhepatic space.
Retained CBD stones will cause pain, jaundice, and high bilirubin level. But it will not cause
collection or manifest any significant inflammatory features unless complicated by cholangitis.

_____________________________________

Q21 - Female patient with incidental finding of gallbladder polyp measuring 0.8 mm and no
stones. She is asymptomatic.
What will you do?

A - Referral for surgery


B - US follow up after 6 months

Answer: B
In the majority of patients, diagnosis of gallbladder polyps is an incidental finding or following
cholecystectomy for acute or chronic cholecystitis. Even though most of the gallbladder polyps
are benign in nature, malignant polyps are found in some cases.

If there are no signs of malignancy, a follow-up with the same modality is done after 6 months. If
by 6 months there is no changes in size, contour, or vascularity, the followup should be
individualized - most often a follow -up after another 12 months could be recommended.
For polyps 6–9 mm in diameter without signs of malignancy, a new US is recommended after 6
months. If this does not show any significant changes a new US is recommended after another 12
months and if no changes is found then no further imaging studies are made. Gallbladder polyps
smaller than 6 mm are not followed up, if there are no features of malignancy.
_____________________________________

58
Q22 - Long scenario of ascending cholangitis. Patient was resuscitated with fluids and antibiotic.
US, gallstones, and dilated CBD
What should be done next?

A - ERCP
B - MRCP
C - Cholecystectomy

Answer: A
Biliary decompression is the mainstay of treatment of ascending cholangitis.

_____________________________________

Q23 - US showed calcified gall bladder without stone asking about management?

A - Cholecystectomy
B - Reassure

Answer:
This is a porcelain gallbladder

For the exam, cholecystectomy.


In real practice, porcelain gallbladder “in itself” is no longer an indication for surgery

_____________________________________

Q24 - A pregnant woman in her 10 weeks with recurrent biliary colicky pain.
When to do her surgery?

Answer:
Traditionally, the recommendation for non-emergent procedures during pregnancy has been to
avoid surgery during the first and third trimesters to minimize the risk of spontaneous abortion
and preterm labor, respectively. This has led some authors to suggest delaying surgery until the
second trimester and that the gestational age limit for successful completion of laparoscopic
surgery during pregnancy should be 26 to 28 weeks. These recommendations are not supported
by good quality evidence; recent literature has demonstrated that pregnant patients may undergo
laparoscopic surgery safely during any trimester without an increased risk to the mother or fetus.
Both laparoscopic cholecystectomy and appendectomy have been successfully performed late in
the third trimester without increasing the risk of preterm labor or fetal demise. Importantly,
postponing necessary operations until after parturition has been shown in some cases, to increase
the rates of complications for both mother and fetus.

_____________________________________

59
Q25 - A man post-cholecystectomy presents with jaundice, fever, and dark urine.
What is the most diagnostic investigation?

A - ERCP
B - US Abdomen
C - CT scan
D - MRCP
E - MRI

Answer:
Is the question about most sensitive diagnostic modality?
ERCP in cases of obstructive jaundice should be strictly reserved for therapeutic intent.
Diagnosis is obtained by US, EUS or MRCP (depending on the clinical scenario)

This topic is confusing to many health care practitioners (not only to interns).

In this clinical setting in hand, you start with transabdominal US. If it shows CBD obstruction,
you proceed for a “therapeutic” ERCP for ductal clearance. If US shows a normal caliber duct,
then the choice will be between EUS and MRCP. They both have a very high diagnostic
accuracy for CBD obstruction. If either of these two modalities show CBD stone, you go for
ERCP. The choice between EUS and MRCP depends on availability and local expertise.

_____________________________________

60
Upper GI surgery
Q1 - Old male patient, smoker, alcoholic, fatigue, debilitated, back and abdominal pain (scenario
didn’t mention jaundice or lab findings).
What is the diagnosis?

A - Acute pancreatitis
B - Chronic pancreatitis
C - Pancreatic Carcinoma
D - Insulinoma

Answer: Incomplete scenario.


There is no acute presentation, so it is not acute pancreatitis
Chronic pancreatitis should have some GI symptoms as well.
Nothing in the question points towards insulinoma.
Smoking and alcohol consumption are well known risk factors for pancreatic cancer, but this is
an answer by exclusion because the question doesn’t give enough clues otherwise.
_____________________________________

Q2 - 34-year-old man presents with slow progressive dysphagia. He has been using H2 blockers
for the last year because of retrosternal discomfort. He has not noticed any weight loss. A
hemoglobin level was done a month ago which reads 13.3 g/dL.
What is the SINGLE most likely diagnosis?

A - Foreign body
B - Plummer-Vinson syndrome
C - Pharyngeal pouch
D - Peptic stricture
E - Esophageal Cancer

Answer: D
_____________________________________

Q3 - Patient with history of pancreatitis two months ago, is now referred to surgery clinic for
mild abdominal pain.
What is the initial investigation?

A - US
B - CT
C - Endoscopy

Answer: A
Initial US.
Imaging modality of choice, CT.

_____________________________________

61
Q4 - Young man presents with multiple episodes of vomiting the last one was bloody. He is
vitally stable (BP 110/76)
What is the most appropriate management?

A - Conservative management
B - Upper GI endoscopy
C - Sengstaken tube

Answer: A
This is a tricky question.
It describes a case of Mallory-Weiss Syndrome. Majority of patients stop bleeding without any
specific intervention; hence, treatment is “conservative”. But you won’t be able to confirm
diagnosis and adopt a conservative approach unless you do an upper GI endoscopy.
So, it is the question of who is first, the chicken or the egg.
I would go for “A” because it is broader than “B”. Conservative management will include things
like IVF, blood transfusion – if needed –, NPO, and a diagnostic EGD.

_____________________________________

Q5 - Post pancreatitis presents with severe mid abdominal pain. He has epigastric tenderness and
high amylase.
What is the most appropriate management?

A - Percutaneous drainage
B - Surgical removal
C - Internal drainage

Answer:
Pancreatic pseudocyst.

Infected, external drainage


Non-infected, internal drainage

Worsening abdominal pain, fever, epigastric tenderness are all features of infection. High white
count is certainly an important surrogate evidence of infection, but it can’t be taken as sole
evidence.

_____________________________________

62
Q6 - Patient with upper GI bleed.
What is the initial next step?

A - Upper endoscopy
B - Sengstaken tube
C - Colonoscopy

Answer: A

_____________________________________

Q7 - 30-year-old male, medically free, presents to ER with several episodes of hematemesis for
the first time. He takes no medications. No family history. Normal abdominal examination, no
guarding, no tenderness.
Hb: 9, Plt: 250, Alk: slightly decreased, all other tests were normal.
What is the diagnosis?

A - Erosive gastritis
B - PUD
C - Mallory-Weiss Syndrome
D - Esophageal varices

Answer: C

_____________________________________

Q8 - Young patient with history of acute pancreatitis 2 weeks ago, presents with epigastric pain.
CT shows 4 cm peri-pancreatic fluid, no necrotic tissues.
What is the management?

A - Internal drainage
B - External drainage
C - Conservative management

Answer: C
Acute pancreatic fluid collections should be just observed. They may resorb or otherwise persist
to form pancreatic pseudocyst in the future (4-6 weeks from the acute attack).

_____________________________________

63
Q9 - Case of intractable pain in chronic pancreatitis, tried trail of NSAIDs and did not manage to
improve his symptoms.
What is the best treatment?

A - Morphine
B - Celiac axis blockade
C - Stenting by ERCP
D - Distal pancreatectomy and splenectomy

Answer: B
Celiac axis blockage

Pain is the most difficult to treat symptom of chronic pancreatitis.


A stepwise strategy is recommended starting with lifestyle modifications such as alcohol
abstinence and low-fat diet, then moving to high dose non-coated pancreatic enzymes and oral
analgesic therapy.
Endoscopic ultrasonography-guided celiac plexus block is safe and well tolerated, with excellent
temporary results in some patients, a significant improvement in pain score occurred in 55% of
the patients.

Patients with debilitating pain, non-dilated pancreatic duct and inflammatory masses may be
candidates for resective surgery.
For patients with dilated main pancreatic duct unresponsive to medical therapy, endoscopy or
decompressive surgery should be considered.

_____________________________________

Q10 - Patient with 2 months history of abdominal pain, failed to respond to medication.
Endoscopy showed multiple antrum ulcerations.
What is the most appropriate management?

A - Partial distal gastrectomy


B - Total gastrectomy
C - Vagotomy with Pyloroplasty

Answer: A
Failure of medical treatment is an indication for surgery.
Antrectomy (resection of the antrum) is the procedure of choice. It is not one of the options,
however, the closest is distal gastrectomy.

_____________________________________

64
Q11 - Patient has no signs or symptoms of cirrhosis all labs normal presents with hematemesis.
Endoscopy showed gastric fundus varices. Imaging, enlarged spleen.

A - Gastritis
B - Mallory-Weiss syndrome
C - Left portal hypertension

Answer: C
Also called, sectoral or segmental portal hypertension.

_____________________________________

Q12 - Patient with epigastric pain. No jaundice. Labs are pending.


US findings: Gallbladder stones, no pericholecystic fluid, the CBD is obscured.
What is the likely diagnosis?

A - Pancreatitis
B - Cholecystitis
C - Obstructive Jaundice
D - Small bowel obstruction

Answer: A
Clinical presentation is very suggestive of acute pancreatitis, awaiting lab confirmation.

_____________________________________

Q13 - Patient went for ERCP or percutaneous transhepatic biliary drain (can’t recall) then
suddenly he developed diffuse emphysema in the abdomen, chest, and neck.
Where is the cause or site of injury?

A - Duodenum
B - Esophagus
C - Gastric

Answer: A
It is ERCP.
Duodenal injury.

_____________________________________

65
Q14 - Patient with severe hematemesis. He is drowsy and disoriented. There is blood coming out
of his mouth.
What do to next?

A - Intubation
B - Octerotide
C - IV fluid

Answer: A
Secure airway.

_____________________________________

66
Bariatric Surgery
Q1 - Patient 1 week post sleeve gastrectomy coming with intermittent fever. On exam abdomen
and chest normal. Wound is clean.
What is the most appropriate step?

A - Reassure
B - CT abdomen

Q2 - Patient 1 week post sleeve gastrectomy coming with persistent vomiting. On exam
abdomen and chest normal. Wound is clean.
What is the most appropriate step?

A - Reassure
B - CT abdomen

Answer: B
This is the time frame of leak. High index of suspicion should be maintained. Liberal use of CT
is recommended, as early detection of leaks impacts the outcome significantly.

_____________________________________

Q3 - What is done before bariatric surgery to help determine the type of procedure?

A - CT scan
B - Endoscopy

Answer: B
This is especially true in patients with reflux symptoms, to determine the degree of esophagitis.

_____________________________________

Q4 - Patient came 3 days after roux-Y surgery complaining of fever, chills, and left shoulder pain
(I think he was stable).
What is the best diagnostic investigation?

A - CT with contrast
B - Endoscopy
C - Laparoscopy
D - Exploratory laparotomy

Answer: C
Likely leak and collection.
Although CT is an option to delineate the site of leak. Diagnostic laparoscopy is the “best”
diagnostic modality.
_____________________________________

67
Q5 - Patient comes 3 days after Roux-en-Y surgery complaining of fever,chills, and left shoulder
pain (BP 90/70 HR :110).
What is the best action for investigation?

A - CT with contrast
B - Endoscopy
C - Diagnostic Laparoscopy
D - US

Answer: C
This is a picture of leak and collection, presenting with features of sepsis as manifested by these
changes on hemodynamics that support the diagnosis.

Patient should be resuscitated with IVF fluids and empiric broad spectrum antibiotics should be
initiated.
Urgent return to OR for diagnostic laparoscopy would be the best course of action.
_____________________________________

Q6 - Patient with DM, BMI 28, tried to lose weight through diet, it didn’t work.
What to do without harming?

A - Orlistat
B - Lorcaserin
C - Phentermine / topiramate
D - Bariatric surgery

Answer:
What I can tell you is that for sure Bariatric Surgery is not indicated here. Rest of options are
“internal medicine”

_____________________________________

Q7 - Patient with progressive abdominal pain post sleeve.


What will you do?

A - Laparoscopy
B - CT
C - Exploration

Answer:
Incomplete scenario
Tens of etiologies can cause abdominal pain. You need to know pain characteristics, associated
symptoms and time frame of its development in relation to his surgery.
_____________________________________

68
Q8 - Post bariatric 3 months with worsening abdominal pain.
What will you do?

A - US
B - Endoscopy
C - Laparotomy

Answer:
Incomplete scenario.
Abdominal pain after bariatric surgeries can be caused by a variety of causes, each has got its
own pattern of presentation and its management line.
Furthermore, not all bariatric procedures are the same when it comes to complications. Some
complications of sleeve are not seen in RYGP and vice versa.

_____________________________________

Q9 - 29-year-old male, 6 days post sleeve gastrectomy, presents to ER with persistent vomiting
since operation. On examination, nothing significant. ABG normal. US, normal.
What is the best management?

A - Reassure
B - Antibiotics
C - Laparotomy

Answer:
US performs very poorly in evaluating possible sleeve leaks.
The gold standard is contrast-enhanced CT with on table oral contrast.
Antibiotic therapy is not indicated in absence of documented infection.
laparotomy is certainly wrong.

_____________________________________

69
Colorectal Surgery

Q1 - Old patient presents with hematochezia and perianal fullness. Examination showed anal
mass 1 cm above the anal verge. Biopsy showed: adenocarcinoma. MRI of the abdomen showed
> 3 cm mass with craniocaudal extension with lymphadenopathy. CT showed no metastasis.
What is the appropriate management?

A - APR
B - LAR
C - Concurrent chemo radiation

Answer: C
Low rectal cancer. Standard of care is neo-adjuvant chemo-rad followed by surgery. It will be
APR in this case.
_____________________________________

Q2 - Male patient, smoker. Has history of appendectomy. His brother has Crohn’s.
What is the risk factor?

A - Male
B - Smoking
C - Family history
D - History of appendectomy

Answer: Not the best question format.


The strongest risk factor for Crohn’s “relapse” is smoking
I think this what the meant by this question
Family history is a strong risk for the “occurrence” of Crohn’s because it runs in families.

_____________________________________

Q3 - Patient with multiple perianal abscess and many fistulas.


How to establish the diagnosis?

A - Pelvic MRI
B - Colonoscopy

Answer: B
Colonoscopy to rule out IBD.

_____________________________________

70
Q4 - Patient with multiple diverticulae in the sigmoid colon and 9 x 12 cm collection.
What is the initial management?

A - Percutaneous drainage
B - Sigmoid resection with anastomosis
C - Diagnostic laparotomy
D - Exploratory laparotomy

Answer: A
Diverticulitis with abscess formation (Henchy ll).

_____________________________________

Q5 - Patient post hemorrhoidectomy comes complaining of severe suprapubic pain and inability
to pass urine.
What is the most likely the cause?

A - Inadequate analgesia
B - Dehydration
C - Urethral injury
D - Hematoma

Answer: A
This real clinical scenario is a clear example of the importance of paying attention to the details.
Pain management is a very important aspect of the overall patient’s experience.
Pain after hemorrhoidectomy causes spasm of pelvic floor muscles, which works exactly like
bladder neck obstruction and that in turn, will manifest as acute urine retention. This is especially
true in people with borderline prostatic enlargement.

Treatment is largely by prevention, by good pain control. If retention develops, it should be


treated with an “in-out, or indwelling urinary catheter”.

_____________________________________

Q6 - A medically free patient comes with a simple anal fistula.


What is the next step in management?

A - Lateral internal sphincterotomy


B - Fistulogram
C - Fistulotomy
D - MRI

Answer: C
_____________________________________

71
Q7 - 25-year-old woman taking steroid for IBD, presents with abdominal pain and bilious
vomiting. On examination, abdomen is distended with right iliac fossa tenderness.
Colonoscopy 2 week ago was normal. Barium study showed a single stricture at terminal ileum,1
cm from ileocecal valve.
Which of the following is the most appropriate management?

A - Stricturoplasty
B - Right hemicolectomy
C - Conservative management
D - Segmental resection with ileostomy

Answer:
There are two types of strictures: Inflammatory and Fibrostenotic.
Stricturoplasty is contraindicated near ileocecal valve.
Right hemi is too much for a young Crohn’s with a short stricture
Segmental resection, can’t be technically done for a lesion 1 cm from ileocecal valve
I will go for conservative treatment

_____________________________________

Q8 - After excision of a pedunculated colonic polyp the result was benign adenoma and patient
has no family history of colon cancer.
What to advice for reduction of colon cancer?

A - Prophylactic sigmoidoscopy
B - Prophylactic colectomy
C - Annual colonoscopy
D - Life style modification (healthy diet and exercise)

Answer: D

_____________________________________

Q9 - What histopathologic type of colorectal polyp has the highest risk of malignancy?

A - Tubulous
B - Tubulovillous
C - Villous

Answer: C
Villous is bad
Tubular is good
Tubulovillous depends on how much of it is tubular and how much villous.
_____________________________________

72
Q10 - Surgery in C3 colon cancer?

A - Curative
B - Palliative
C - Diagnostic

Answer: I’m sure it is stage three what you mean.

Stage III colon cancers have spread to nearby lymph nodes, but they have not “yet” spread to
other parts of the body.
Surgical resection of the affected part of the colon along with nearby lymph nodes, followed by
adjuvant chemo is the standard treatment for this stage.

Sage III colon cancer has about a 40 percent chance of cure (compared to stage IV that has only
a 10 percent chance of cure). Chemotherapy is used after surgery in many colon cancers which
are stage II, III, and IV as it has been shown that it increases the survival rates.

_____________________________________

Q11 - Old male patient presents with lethargy and weight loss for two months. Examination is
notable for pallor and second-degree hemorrhoids. Lab showed low hemoglobin.
What is the most likely diagnosis?

A - Rectal cancer
B - Cecal cancer
C - Hemorrhoids
D - Sigmoid cancer

Answer: B
Although rectal and sigmoid cancer can produce anemia, they usually present with PR bleeding
Hemorrhoids rarely (if ever) cause anemia
Cecal cancers bleed slowly but surely to the bowel lumen and blood gets mixed with stool, so
patients don’t see blood per rectum, but if stool is tested for occult blood it is going to be
positive.

_____________________________________

73
Q12 - 25-year-old man presents to ER with severe pain during and after defecation for 3 days
associated with passage of a small amount of fresh blood after defecation. Physical examination
confirms an acute posterior anal fissure. Digital and proctoscopic examinations were not
performed due to anal pain.
Which of the following is the most appropriate management?

A - Examination under anesthesia


B - Lateral internal anal sphincterotomy
C - Chemical sphincterotomy with diltiazem
D - Botulinum toxin paralysis of anal sphincter

Answer: C
An ideal recall

EUA wouldn’t add anything since diagnosis has already been made.

Surgical option would be very immature since a good percentage of acute fissures respond to
medical treatment.

Botox injection, though used in certain indications, is still off-label.

_____________________________________

Q13 - What type of hemorrhoids is treated with sclerotherapy?

A - Internal
B - External
C - Prolapsed
D - Thrombosed

Answer: A
Not a standard treatment.

_____________________________________

74
Q14 - 24-year-old woman with recurrent on-off PR bleeding after defecation. She has
spontaneously reduced hemorrhoids. Hb was low. On examination: 3 hemorrhoids at 2,7,10
O’clock
How would u manage?

A - Observation
B - Band ligation
C - Hemorrhoidectomy
C - Conservative

Answer: B
Please note that anemia is not a usual presentation of hemorrhoids.
PR bleeding causing anemia should prompt an active search for a more genuine pathology, even
in presence of hemorrhoids.
Hemorrhoids that reduce spontaneously is Grade ll. Like grade l, grade ll hemorrhoids are
managed conservatively, plus band ligation for grade ll.

_____________________________________

75
Q15 - Young patient on-off bleeding PR from a grade II hemorrhoid (spontaneously returning), I
think patient also has low Hb.
What is the management?

A - Conservative
B - Angio something
C - Band ligation
D - Traditional hemorrhoidectomy

Answer: C

_____________________________________

Q16 - An elderly man comes with perfuse PR bleeding of bright fresh blood. There was no
weight loss, abdominal pain, or any other symptoms.
What is the most likely cause of this presentation?

A - Colon cancer
B - Internal piles
C - Angiodysplasia
D - Diverticulosis

Answer: D
Diverticular disease and angiodysplasia are the main two differentials for such presentation.
Diverticular disease, however, is more frequent.

_____________________________________

Q17 - Diarrhea with recurrent perianal abscess, and multiple fistulas


What investigation to do?

A - Colonoscopy
B - Fistulogram

Answer: Colonoscopy to role our Crohn’s.

_____________________________________

76
Q18 - Case of paralytic ileus in a patient using antipsychotic agents. He is vitally stable and has
no sign of peritonitis. X-ray shows dilated colon >10 cm. CT, no mechanical obstruction.
What is the most appropriate management?

A - Total colectomy
B - Partial colectomy with stoma
C - Decompressive rectal tube

Answer: C
This is not paralytic ileus.
This is acute colonic pseudo-obstruction (Ogilvie Syndrome)
Treatment is by controlling precipitating factors, administration of Neostigmine and / or insertion
of rectal tube under sigmoidoscopic guidance.

_____________________________________

Q19 - A patient with colon cancer complicated by colovesical fistula is seen for recurrent UTI,
pneumaturia and passage of fecal matters with urine.
What is the most appropriate management?
A - Conservative treatment
B - Bladder repair and resection

Answer: B
Remember your F.R.I.E.N.D.S.
Neoplasm (N) will not allow fistula to heal on conservative treatment
Go for resection of that segment of the colon along with en-block resection of bladder wall.

_____________________________________

Q20 - 57-year-old woman has third degree hemorrhoids (with NO bleeding)


Which of the following is the most appropriate management?

A - High fiber supplementation


B - Rubber band ligation
C - Surgical hemorrhoidectomy

Answer: C
Though rubber banding can be use in a certain proportion of grade lll hemorrhoids, surgical
excision remains the mainstay of treatment in majority of cases.

_____________________________________

77
78
Q21 - 45-year-old women with no family history of cancer asks you when to screen colon
cancer?

A - Screen now
B - After 5 years
C - No need
D - After 10 years

Answer: B
Screening for colorectal cancer for average risk individuals begins at the age of 50. Colonoscopy
is the standard screening method, if normal, you do it at 10 years interval. Other screening
modalities – with varying sensitivities – exist.

_____________________________________

Q22 - 38-year-old man diagnosed with ulcerative colitis for 6 years, presents with perianal pain.
When to screen for colorectal cancer?

A - 8 years after diagnosis


B - Now since he is symptomatic

Answer: A
Risk of CRC increases after 8 years of disease activity.
_____________________________________

Q23 - 47-year-old female presents with iron deficiency anemia. She has 3rd degree hemorrhoids.
Which of the following is the most appropriate thing to do?

A - Colonoscopy
B - Hemorrhoidectomy
C - CT scan

Answer: A

_____________________________________

79
Q24 - An old male patient was admitted as a case of large intestinal obstruction. He underwent
rigid sigmoidoscopy that showed a mass in the sigmoid region. A biopsy was taken and came
back as adenocarcinoma.
What is the best next step?

A - Colonoscopy
B - CT abdomen
C - MRI pelvis
D - Sigmoidectomy

Answer: D
This is a patient who is currently obstructed.
Colonoscopy is not possible in an obstructed patient.
CT, although important for staging, is not expected to change management in the acute setting.
MRI has a role in evaluating rectal cancer but not sigmoid cancer.

With all my due respect to whoever wrote this question, it is extremely difficult to imagine
having a patient with large bowel obstruction and yet we have the luxury of doing
sigmoidoscopy, obtaining biopsy, and waiting for its result that takes several days.

Bottom line, this obstructed patient needs sigmoidectomy.

_____________________________________

80
Q25 - Interval of colonoscopy for a polyp size of 0.9 cm and results of histopathology is
adenoma?

A - 3-6 months
B - 1 year
C - 3 year
D - 10 years

Answer: D
The following recommendations for post-polypectomy colonoscopic surveillance apply to all
patients who had one or more polyps that were completely removed during a high quality
baseline colonoscopy.

1 ESGE recommends that patients with complete removal of 1 – 4 < 10 mm adenomas with low
grade dysplasia, irrespec- tive of villous components, or any serrated polyp <10mm without
dysplasia, do not require endoscopic surveillance and should be returned to screening.

2 ESGE recommends surveillance colonoscopy after 3 years for patients with complete removal
of at least 1 adenoma ≥ 10 mm or with high grade dysplasia, or ≥ 5 adenomas, or any serrated
polyp ≥ 10 mm or with dysplasia.

3 ESGE recommends a 3 – 6-month early repeat colonoscopy following piecemeal endoscopic


resection of polyps ≥20mm.
A first surveillance colonoscopy 12 months after the repeat colonoscopy is recommended to
detect late recurrence

4 If no polyps requiring surveillance are detected at the first surveillance colonoscopy, ESGE
suggests performing a second surveillance colonoscopy after 5 years.
Weak recommendation, low quality evidence.
After that, if no polyps requiring surveillance are detected, patients can be returned to screening.

5 ESGE suggests that, if polyps requiring surveillance are detected at first or subsequent
surveillance examinations, surveillance colonoscopy may be performed at 3 years.

_____________________________________

81
82
Q26 - After resection of colon adenocarcinoma with clear margins, what is the recommended
follow up?

_____________________________________

Q27 - Anal examination shows fistulas at 3 and 9 o’clock.


What is the next step?

A - Colonoscopy
B - Reassurance

Answer: A
Complex fistula. Need to rule out Crohn’s.

_____________________________________

83
Q28 - A patient with Crohn’s disease treated with azathioprine presented complaining of the pus
out from anal. Vital signs stable also no fever.
What is the next step?

A - Percutaneous drainage
B - Broad IV antibiotic
C - Swab and culture

Answer:
Pus coming from within, or around anal canal comes either from a perianal fistula or a partially
drained perianal abscess.
Ideal answer would be Examination Under Anesthesia (EUA).
Starting systemic antibiotics is shooting in the dark. You don’t know what you are treating.

Percutaneous drainage is something you do when you have a deep collection (intraabdominal or
pelvic). These collections don’t present with pus from the anus.

What we are left with is swab and culture. Although it is important to guide antibiotic therapy, it
doesn’t address the underlying pathology (be it a fistula or an abscess).

So, I hope option D would be EUA.

Also note that the question talks about “pus” and not about mucus discharge which can be a
manifestation of disease activity. So, increasing the dose of Azathioprine “which appeared in
another recall” is not an option either.

_____________________________________

Q29 - Patient underwent sigmoidoscopy found to have a mass that turned out it to be adenoma.
What will you do next?

A - CT scan
B - Sigmoidectomy
C - Colonoscopy

Answer: C
Sigmoidoscopy screens part of the colon only.
You need to check the rest of the colon for synchronous adenomatous polyps

_____________________________________

84
Q30 - Patient underwent sigmoidoscopy and found to have multiple masses in the distal sigmoid,
histopathology was adenocarcinoma.
What will you do next?

A - Colonoscopy
B - CT abdomen
C - Sigmoidectomy
D - MRI

Answer: A
Same thing
You need to check the rest of the colon for any synchronous lesions whether adenomas or
carcinomas.

CT will certainly be needed for staging

Sigmoid colectomy is the surgical treatment of choice once rest of the colon is checked and no
further lesions are found.

MRI has no role in management of sigmoid cancer (as opposed to rectal ones)

But everything starts by evaluating the entirety of the colon.

_____________________________________

85
Q31 - 40-year-old male known case of schizophrenia and depression. Presents to ER with
recurrent abdominal pain and distension. Examination shows significant abdominal distension.
X-ray shows hugely dilated colon with a diameter of 10 cm.
What is the most appropriate management?

A - Barium enema
B - Emergency colectomy
C - Diversion of left colon
D - Decompression colonoscopy with rectal tube

Answer: D
Acute colonic pseudo-obstruction (Ogilvie's syndrome) is a disorder characterized by acute
dilatation of the colon in the absence of an anatomic lesion that obstructs the flow of intestinal
contents.
Acute colonic pseudo-obstruction usually occurs in hospitalized or institutionalized patients in
association with a severe illness or after surgery and in conjunction with a metabolic imbalance
or administration of culprit medications. In a large, retrospective series that included 400 patients
with acute colonic pseudo-obstruction, the most common predisposing conditions were
nonoperative trauma, infection, and cardiac disease, each of which were associated with 10
percent of cases. In this series, cesarean section and hip surgery were the most common surgical
procedures associated with acute colonic pseudo-obstruction. In a systematic review of 125,
postpartum cases of acute colonic pseudo-obstruction, 62 (92 percent) occurred following
caesarian section. Patients with a cecal diameter of >12 cm are at extremely high risk of
perforation.

Acute colonic pseudo-obstruction usually involves the cecum and right hemicolon, although
occasionally colonic dilation extends to the rectum. Acute colonic pseudo-obstruction appears to
be more common in men and in patients over the age of 60 years. However, cases have been
reported in children. Acute colonic pseudo-obstruction is a rare complication of surgery,
occurring in 0.06 percent of patients after cardiac surgery, 0.29 percent of burn patients, and 0.7
to 1.3 percent of patients after orthopedic surgery.

_____________________________________

86
Q32 - A 36-year-old male known case of Crohn’s for 10 years, presents to ER with abdominal
pain, fever, vomiting, and diarrhea, On examination, there is abdominal tenderness.
CT showed: 12×15 collection and ileo-jejunal fistula.
How to manage?

A - Laparoscopic drainage
B - Percutaneous drainage
C - Open drainage
D - Open drainage with fistula resection.

Answer: B
Percutaneous drainage, IV antibiotics, nutritional support and medical control of disease activity.
Surgery for fistula resection should be deferred till flare up of the disease is brought under
control.
_____________________________________

Q33 - Patient underwent lumbar spine surgery through back approach. On the 4th post op day, he
was seen by GS due to severely distended abdomen with RIF tenderness (or mass not sure).
Barium enema showed huge distention involving the whole bowel (with cecal diameter of
13cm), but there was no extravasation or gas in the peritoneum.
Which of the following most appropriate management?

A - Observation
B - Colonoscopy
C - Cecostomy
D - Total colectomy

Answer: B
Acute colonic pseudo-obstruction (ACPO), known as Ogilvie’s Syndrome, is a distinct form of
colonic dilatation occurring in the absence of underlying mechanical or anatomic etiology. It is
most encountered in older adults with multiple underlying comorbidities but may also develop in
otherwise healthy patients after a traumatic injury or following a surgical operation.
It is important to remember that the diagnosis of ACPO is one of exclusion and that more
common causes of functional or mechanical bowel dilatation must be investigated. Symptoms
and signs of the disease usually manifest over 3 to 5 days but may also develop more acutely,
sometimes within 48 hours. ACPO is considered complicated when the patient develops any
evidence of bowel ischemia, peritonitis, or perforation; the risk of complication increases directly
with increasing cecal diameter and duration of illness.
The primary goal of treatment is urgent bowel decompression. Treatment options include direct
pharmacologic interventions (neostigmine) and endoscopic therapies (decompression via
sigmoidoscopy or colonoscopy with placement of rectal tube)
Surgical interventions are reserved for those failing endoscopic procedures, or for patients who
develop ischemia or perforation.
_____________________________________

87
Q34 - Old patient with signs and symptoms of bowel obstruction. He has left sided tenderness
(hypochondrium to ileac), and a mass felt at left lumbar region.
X ray: dilated large bowel pointing to left corner of abdomen.
What is the site of volvulus?

A - Sigmoid
B - Transverse
C - Cecum
D - Ascending colon

Answer: C
Cecal volvulus accounts for ~10% of all intestinal volvulus, and generally occur in somewhat
younger patients than with a sigmoid volvulus, most being 30-60 years old. There are two
predisposing factors that are important for the development of a cecal volvulus:
developmental failure of peritoneal fixation allows the proximal colon to be free and mobile: this
occurs in 11-25% of the population
restriction of the bowel at a fixed point within the abdomen, acting as a fulcrum for rotation, e.g.
adhesions, abdominal mass, scarring from calcified lymph nodes
Medical history of these patients may include prior abdominal surgery, the presence of a pelvic
mass, violent coughing, atonia of the colon, extreme exertion, unpressurised air travel, or third-
trimester pregnancy.
Cecal volvulus presents with clinical features of proximal large bowel obstruction. This is
usually with colicky abdominal pain, vomiting, and abdominal distension.

_____________________________________

88
89
Q35 - Patient has annular ulcer 10 cm from anal verge.
What to do next?

A - Anterior resection
B - Colonoscopy
C - Resection and anastomosis

Answer:
Colonoscopy for pathology

This is typical solitary ulcer syndrome


But the question is not right. How did we know about the ulcer and it’s exact description if we
haven’t done a colonoscopy in the first place?

The answer for this question as it is now, colonoscopy.

_____________________________________

Q36 - Patient with pedunculated polyp in sigmoid colon, resected and histopathology showed,
well differentiated adenocarcinoma involving only muscularis mucosa with free margins.
What to do?

A - FU
B - Sigmoidectomy
C - Fulguration of the polyp site
D - Partial colectomy

Answer: A

This is carcinoma in-situ, completely excised with free margins. Needs surveillance.

_____________________________________

Q37 - Women 54 age, what screening recommendations regarding colon cancer?

A - Occult blood annually


B - Colonoscopy every 5 years

Answer: A
Gold standard screening modality for colorectal cancer is colonoscopy every 10 years, starting at
the age of 50, (unless abnormalities detected)

Annual fecal occult blood test is a cheaper alternative with less accuracy.

_____________________________________

90
Q38 - Typical scenario of diverticulitis (LLQ pain, fever). CT showed 9 x 10 cm collection with
thick edematous sigmoid wall and multiple diverticula.
What is the most appropriate management?

A - Sigmoidoscopy
B - Percutaneous drainage
C - Sigmoidectomy with primary anastomosis

Answer: B
Drainage of the collection percutaneously, along with bowel rest and systemic IV antibiotic
therapy, is all that is needed in the acute setting

_____________________________________

Q39 - A patient presents to the clinic with perianal discharge. No abdominal pain or PR bleeding.
Examination showed three fistulas were noted at the 3 5 7o'clock positions. Proctoscopy was
clear.
What is the most appropriate next step?

A - Colonoscopy
B - MRI
C - Fistulogram
D - US

Answer: A
Colonoscopy to rule out Crohn’s.

_____________________________________

Q40 - Old female, postmenopausal. Complains of fatigue, tiredness, and palpitation. Labs shows
anemia.
What is the best next step?

A - FOBT
B - Upper and lower endoscopy

Answer: B

_____________________________________

91
Q41 - 67-year-old woman with weight loss and rectal bleeding. Rectal examination showed
blood clots but no masses
What is the most appropriate management?

A - Colonoscopy
B - Proctoscopy

Answer: A
Presence of local causes of bleeding (like hemorrhoids) can’t explain weight loss and can’t rule
out colorectal neoplasms
Colonoscopy is the way to go
_____________________________________

Q42 - Anal pain, discharge staining the underwear. In palpation, there is tenderness above
sphincter.
What is your diagnosis?

A - Perianal fistula
B - Inter-sphincter abscess

Answer: B
For a fistula you need to find an external opening
_____________________________________

Q43 - Patient presents with small bowel obstruction. Imaging showed 3 strictures on the ilium.
What is the most likely diagnosis?

A - Crohns
B - GI Stromal tumor
C - Intestinal lymphoma

Answer: A
_____________________________________

Q44 - Was young patient 14-year-old I think, colonoscopy found bleeding coming from ileocecal
valve?

A - Crohn’s
B - Angiodysplasia
C - Mickles diverticulum

Answer: C
It is the most common cause of lower GI bleeding in this age group.
_____________________________________

92
Q45 - A patient known to have Crohn’s disease treated with azathioprine presented complaining
of pus coming out from the anus. Vital signs are stable and there is no fever.
What is the next step?

A - Percutaneous drainage
B - Broad spectrum IV antibiotics
C - Swab and culture

Answer: This question is problematic.


It describes “pus” coming out of the anus in a Crohn’s patient. Possibilities are perianal fistula or
abscess. Ideally, this patient should go for Examination Under Anesthesia (EUA). This will
reveal the exact underlying pathology.

Now let’s examine the options provided:


A - percutaneous drainage, though the treatment of choice of deep (intraabdominal or pelvic)
collections, is not an option here because there is nothing to drain.

B - Antibiotics
This is very likely needed in case of abscess, but it is not the mainstay treatment.

C - Swab for culture.


Would be done after EUA to help guide antibiotic therapy

Bottom line, the question as it is now can’t be answered. There is obviously a missing option
here, option D. Let’s hope that they will put it as EUA

_____________________________________

Q46 - Patient with abdominal pain, imaging shows: increased thickness of sigmoid and thumb
print sign due to edema.
What is the next step?

A - Colonoscopy
B - Diagnostic laparoscopy
C - Exploratory laparotomy

Answer: A
Infectious colitis.
_____________________________________

93
Q47 - Patient presents with perianal pain and fever. On exam you found posterior fullness with
minimal discharge.
What is the most likely diagnosis?

A - Abscess
B - Perianal fistula

Answer: A

_____________________________________

Q48 - 24-year-old male complaining of tender mass and redness in perianal area. He has history
of recurrent presentations that end with spontaneous drainage. Abdominal examination normal.
Perianal examination shows one fistula with redness.
What is the appropriate management?

A - CT pelvis
B - Oral antibiotics with outpatient follow-up
C - Needle aspiration
D - Examination under anesthesia

Answer: D
Although this patient would need to be worked up for Crohn’s, this acute presentation is very
suggestive of abscess. EUA, followed by incision and drainage would be the way to go.

_____________________________________

Q49 - A 15-year-old boy is very concerned that he may be developing the same disease as his
father who died aged 37 y/o of colon cancer. The boy has noticed he has developed unexpected
episodes of diarrhea and lower abdominal pain. These patients also develop other comorbidities.
Which of the following would be a typical example?

A - Astrocytoma
B - Osteosarcoma
C - Retinoblastoma
D - Duodenal carcinoma

Answer: D
Periampulary cancer
They have to go for surveillance upper GI scopes
I would choose duodenal cancer.

_____________________________________

94
Q50 - Patient after appendectomy found to have 0.5 mm carcinoid tumor in mesoappendix.
What's your action?

A - Right hemicolectomy
B - Follow-up
C - Chemotherapy
D - Appendectomy

Answer: A

_____________________________________

Q51 - UC increases risk of colon cancer in?

A - Duration of disease more than 8 years


B - Extension beyond splenic flexure
C - If it was associated with primary sclerosing cholangitis
D - Family history of colorectal cancer

Answer:
Both A and C are correct
I would choose A for one reason: it is an exam at interns’ level

UC with PSC increases the risk hit CRC by three times.

_____________________________________

95
Surgical Oncology

Q1 - An old female for elective cholecystectomy. During procedure, the surgeon noticed a mass
in stomach.
What is the best action?

A - Cholecystectomy only
B - Resection of the mass with a safety margin
C - Take a biopsy from the gastric mass
D - Partial gastrectomy

Answer: A
Incidentaloma, very common exam question. Principle is simple, do what you are there to do.
Document your findings and do the appropriate work-up and management afterwards.
So, in this scenario, do your cholecystectomy, examine the abdomen for presence of peritoneal
metastasis, ascites, or visibly enlarged lymph nodes. Post-operatively, arrange for EGD and
imaging.

_____________________________________

Q2 - 60-year-old patient complaining of dull aching swelling non tender found to have a
retroperitoneal mass 20×20 cm in size, and multiple masses in the liver.
What is the most likely diagnosis?

A - Liposarcoma
B - Germ cell tumor
C - Neural tube something
D - Lymphosarcoma

Answer: A
Liposarcoma.
Most common sarcoma subtypes are the well-differentiated liposarcoma
and dedifferentiated liposarcoma, followed by the leiomyosarcoma.
Surgery is the mainstay of treatment of non-metastatic retroperitoneal liposarcoma

Metastatic germ cell tumors to the liver are rare and usually carry adverse outcomes and mainly
occur in young age group.
_____________________________________

96
Q3 - A case of chronic Hepatitis C presents with RUQ mass. Investigations show 6 x 6 cm
hepatocellular carcinoma.
What is the best management?

A - Chemotherapy
B - Radiotherapy
C - Transcatheter arterial chemoembolization (TACE)
D - Surgical resection

Answer: D
Surgical resection Is the best management.

Surgical Resection is considered the first-line treatment for patients with solitary tumors
confined to the liver without radiographic evidence of invasion of the vasculature and preserved
liver function

Several treatment modalities are available, but only surgical resection and OLT are curative
treatment.

OLT is available only for Milan criteria.


Other treatment options include radiofrequency ablation, microwave ablation, percutaneous
ethanol injection, transarterial chemoembolization, radioembolization (TACE), cryoablation,
radiation therapy, stereotactic radiotherapy, systemic chemotherapy, and molecularly targeted
therapies.

_____________________________________

97
98
Q4 - What is the most accurate modality to determine “T” stage of gastric Adenocarcinoma?

A - Endoscopic US
B - Transabdominal US
C - Gastrograffin

Answer: A
Same question comes up for esophageal carcinoma.
The answer is the same, EUS.

_____________________________________

Q5 - 45-year-old patient has a 5 cm mass in the right upper extremity. MRI shows it to arise from
triceps.
What is next step in management?

A - Incisional biopsy
B - Excisional biopsy
C - Core biopsy

Answer: C
Core biopsy

Excisional biopsies are not routinely recommended. An excisional biopsy may be the most
practical option for superficial lesions that are <5 cm in size.
Incisional or open biopsy may be an option in selected cases where repeated core biopsies have
failed to achieve a diagnosis.

Core biopsy is less invasive and can be done under local anesthesia and the material obtained
from a core biopsy is superior (in amount and viability) compared to that obtained by fine needle
aspiration cytology (FNAC).

_____________________________________

99
Q6 - Mid-thigh lump with normal overlying skin and positive fluctuating test.
What is the diagnosis?

A - Lipoma
B - Sarcoma
C - Aneurysm
D - Sebaceous cyst

Answer: A
Lipomas are defined as a common subcutaneous tumor composed of adipose (fat) cells, often
encapsulated by a thin layer of fibrous tissue. In fact, these are the most frequently encountered
neoplasms by clinicians. The tumors typically lie in the subcutaneous tissues. The masses are
often benign, there is usually no reason for treatment. They pose no threat to the patient unless
they are uncomfortable due to being located on joints or rapidly growing, which is uncommon,
as the typical lipoma growth is slow.

_____________________________________

Q7 - Known alcoholic for long time, presents with enlarged lymph node in mid cervical region.
What is the best next step?

A - Laryngoscopy
B - Excisional biopsy
C - Needle biopsy

Answer: C
FNA..

_____________________________________

Q8 - Male, alcoholic presents with fever and weight loss. Imaging shows cirrhosis and multiple
liver lesions?

A - HCC
B - Liver cyst
C - Liver abscess
D - Pancreatic cancer

Answer: C
Among these options, the only one that can cause fever is liver abscess.

_____________________________________

100
Q9 - What is the most common small bowel malignancy?

A - Adenocarcinoma
B - Carcinoid

Answer: A
This is where people get it wrong.
Small bowel is the commonest site of occurrence of carcinoid tumor, but - like the rest of GI
tract - adenocarcinoma is the most common malignancy of small bowel.

_____________________________________

Q10 - Muscle tumor in the triceps and looks malignant.


How to diagnose?

A - Incisional biopsy
B - Excisional biopsy
C - Core needle biopsy

Answer: C
Core biopsy
Excisional biopsies are not routinely recommended. An excisional biopsy may be the most
practical option for superficial lesions that are <5 cm in size.
Incisional or open biopsy may be an option in selected cases where repeated core biopsies have
failed to achieve a diagnosis.
Core biopsy is less invasive and can be done under local anesthesia and the material obtained
from a core biopsy is superior (in amount and viability) compared to that obtained by fine needle
aspiration cytology (FNAC).

_____________________________________

101
Q11 - Elderly with new onset of vague abdominal pain, jaundice, and generalized fatigue. CT
scan showed liver masses involving 70% of the liver volume.
What is your next step?

A - Colonoscopy
B - Endoscopy
C - Diagnostic laparoscopy

Answer: A
Colonoscopy to rule out colorectal cancer which is the most common source of liver metastasis
25% of colorectal cancers present with liver mets.

You need to understand what the question wants. This question clearly wants to test your
knowledge about behavior of colorectal cancers.

You may also find in the options:


CEA level or CT chest. All are needed at some point. But this is not what the question wants to
test.

_____________________________________

Q12 - Old patient with chronic hepatitis B. CT scan showed liver masses involving 70% of the
liver volume.
What is your next step?

A - Colonoscopy
B - Chemotherapy
C - Biopsy

Answer: C
Biopsy.
Hepatitis B virus has been implicated in the cause of up to 80% of cases of hepatocellular
carcinoma (HCC)
Alpha feto-protein and biopsy will differentiate between regenerative nodules or HCC

As opposed to a previous recall where we have chosen colonoscopy to look for primary tumor
metastasizing to the liver, this current question talks about liver lesions in a Hepatitis patient.
Here HCC is your principal diagnosis. Go for biopsy.

_____________________________________

102
Q13 - A female known case of Hepatitis C virus has multiple liver masses.
What is your further management?

A - Liver biopsy
B - US
C - CT

Answer: A
_____________________________________

Q14 - 68-year-old patient was recently diagnosed with hepatocellular carcinoma.


What is the next step?

A - Colonoscopy
B - Chest CT
C - CT liver
D - Surgery

Answer: B
Chest CT for staging
The lung, abdominal lymph nodes, and bone are the most common sites of extrahepatic
metastatic HCC.

_____________________________________

Q15 - Old patient with esophageal cancer, left supraclavicular LN metastasis, and absolute
dysphagia.
What is the treatment?

A - Palliative Chemo
B - Palliative stent
C - Esophagectomy and gastrostomy
D - Resection and anastomosis

Answer: B

_____________________________________

103
Q16 - Long scenario of smoking and obese patient. At imaging there is 4 x 4 cm liver mass.
What is the most important thing to advise the patient?

A - Decrease high carbohydrate and fatty meals


B - Stop smoking
C - Eat diet rich in fiber
D - Eat high protein diet

Answer: A

Nonalcoholic fatty liver disease (NAFLD) is a term for a range of liver conditions caused by a
build-up of fat in the liver. It's usually seen in people who are overweight or obese.

NAFLD is an aggressive form of fatty liver disease, which is marked by liver inflammation and
may progress to advanced scarring (cirrhosis) and liver failure

NAFLD and NASH are both linked to the following:


* Overweight or obesity
* Insulin resistance
* High blood sugar m
* High levels of fats, particularly triglycerides, in the blood

To reduce your risk of NAFLD:


* Choose a healthy diet. Choose a healthy plant-based diet that's rich in fruits, vegetables, whole
grains, and healthy fats.
* Maintain a healthy weight. If you are overweight or obese, reduce the number of calories you
eat each day and get more exercise. If you have a healthy weight, work to maintain it by
choosing a healthy diet and exercising.

_____________________________________

Q17 - Patient with non-ACTH dependent Cushing for adrenalectomy.


What is the management?

A - Postoperative fludrocortisone
B - Postoperative metatone
C - Preoperative hydrocortisone
D - Postoperative hydrocortisone

Answer: C
_____________________________________

104
Q18 - 59-year-old male complaining of fatigue palpitation and shortness of breath. Examination
showed pallor. CBC suggestive of iron deficiency anemia. Besides giving iron supplement.
What investigation should be requested?

A - Urea breath test


B - Fecal occult blood test
C - Endoscopy and colonoscopy

Answer: C
Gastric and colorectal malignancies should be ruled out.

_____________________________________

105
Q19 - Patient was thought to have appendicitis and underwent appendectomy. Pathology showed
carcinoid tumor 1 cm in size.
What is the most appropriate management?

A - Follow up
B - Chemo
C - Radiation
D - Right hemicolectomy

Answer: A
Questions about carcinoid tumors of the appendix are very popular and they frequently appear in
SMLE.
They may change size, location, or histopathologic characteristics.

You find here a compilation of all pertinent guidelines:

NCCN recommendations for appendix NETs are as follows:


• Tumors ≤2 cm confined to the appendix: Appendectomy
• Tumors ≤2 cm with lymphovascular or mesoappendiceal invasion or atypical histologic
features: More aggressive treatment can be considered
• Incomplete resection or tumors >2 cm: Staging with abdominal/pelvic CT or MRI; if no distant
disease, re-exploration with a right hemicolectomy

The NANETS guidelines include the following recommendations:


• Excision for tumors ≤2 cm; consider right hemicolectomy with node dissection if high- risk
features are present
• Tumors >2 cm: Right hemicolectomy with node dissection

The 2016 ENETS revised guidelines recommendations include:


• Tumors ≤2 cm: simple appendicectomy unless incompletely resected
• Right hemicolectomy with node dissection only in rare tumors measuring 1–2 cm but with
positive or unclear margins or with deep mesoappendiceal invasion (ENETS T2), higher
proliferation rate (G2) and/or vascular invasion
• Tumors >2 cm: right hemicolectomy with node dissection.

_____________________________________

106
Q20 - 70-year-old patient presents with sudden abdominal tenderness. US normal and normal
examination. HB low.
Asks about what next?

A - Occult stool
B - Endoscopy and colonoscopy

Answer:
Although this lady will need upper and lower GI scopes in search of a source of her anemia,
none of these two modalities has a role in evaluating this acute presentation of pain and
tenderness. Same applies to fecal occult blood testing.
_____________________________________

Q21 - Which lymph node indicate malignancy?

A - Cervical
B - Submandibular
C - Supraclavicular

Answer: C
Virchow's node is a lymph node and is a part of the lymphatic system. It is the thoracic duct
end node. It receives afferent lymphatic drainage from the left head, neck, chest, abdomen,
pelvis, and bilateral lower extremities, which eventually drains into the jugulo-subclavian venous
junction via the thoracic duct.
"Troisier sign" describes an enlarged palpable hard left supraclavicular node following Charles-
Emile Troisier's work, which is now sometimes used interchangeably with Virchow's node.

Numerous studies have shown Virchow's node to be of clinical significance, especially


concerning malignancies. Due to its lymphatic function, the Virchow's node is a potential
seeding site for not only gastrointestinal malignancies, but also pulmonary
adenocarcinoma, prostate cancer, lymphoma, and ovarian cancer, among others.
_____________________________________

Q22 - Patient with pancreatic cancer presents with lower limb pain that is edematous tender.
What is the most likely diagnosis?

A - Becker cyst rupture


B - Cellulitis
C - DVT

Answer: C
Malignancies increase the risk of thromboembolic events as they are associated with
hypercoagulable state.
_____________________________________

107
Peri-Operative Care
Q1 - Patient admitted for DVT and was given enoxaparin (LMWH). During the same admission,
she developed PE. Her vitals are stable.
What to do?

A - Continue same management


B - Switch to sodium heparin

Answer:
I will go for the same management.
LMWH is more effective, and its behavior is more predictable in terms of absorption and drug
level.
In addition, with IV heparin, it is frequently difficult to achieve consistent and therapeutic PTT
level.
_____________________________________

Q2 - Obese female using OCPs underwent a major abdominal surgery few days ago presents
with unilateral calf swelling with pain and tenderness.
What investigation will you order?

A - Doppler US
B - Chest CT
C - CT Angio
D - Angiography

Answer: A
Obesity, OCP and recent surgery are all risk factors for DVT

_____________________________________

Q3 - Patient was discharged after laparotomy and readmitted with retained sponge in the
abdomen.
What is the best technique to avoid this complication?

A - Time out by a nurse


B - Sponge count during surgery
C - Xray after high-risk procedures

Answer:
Time out is a very important quality measure, but it has nothing to do with sponge or instruments
count.
X-ray is needed once a missing sponge is suspected
The answer is sponge count by TWO nurses (scrub nurse and circulating one).

_____________________________________

108
Q4 - Patient 10 days following sleeve gastrectomy develops SOB and diminished popliteal pulse.
What is the next step of management?

A - TPA
B - Heparin
C - IVC filter
D - Aspirin

Answer: B
DVT and PE. Start systemic anticoagulation by IV heparin even before obtaining diagnostic
imaging (doppler US for the lower extremities and chest CT).

_____________________________________

Q5 - Which of the following prevents post-operative paralytic ileus?

A - Avoid early postoperative feeding


B - Start early postoperative feeding

Answer: B
Feeding enhances bowel motility

_____________________________________

Q6 - Patient post-appendectomy, presents to the clinic with 20 ml seroma from his surgical
wound.
What is the appropriate management?

A - Regular dressing
B - Wound exploration
C - Antibiotics
D - CT scan

Answer: A
In absence of features of infection, “prophylactic” antibiotics have no role.

CT scan would be ordered for search of deep collections if clinically suspected based on
patients’ symptoms (the question doesn’t tell us that the patient has any other complaint)

Wound exploration would be needed to evacuate pus in case of wound infection.


Wound seroma doesn’t need antibiotics or evacuation. All that is needed is dressing to keep the
wound clean and dry and to observe for resolution.

_____________________________________

109
Q7 - Surgeon diagnosed a patient with appendicitis and took him for appendectomy.
Intraoperatively, appendix was normal. Then doctor removed the appendix.
What is the most appropriate action with regards to this decision?

A - Tell the patient that appendix was normal and was removed according to guidelines
B - Don't tell patient
C - Call another surgeon to tell him
D - Inform the ethical committee

Answer: A

This is a common scenario. A patient presents with enough signs and symptoms of acute
appendicitis to justify operative exploration then, intraoperatively, proves to have a normal
appendix. The surgeon should start an active search for an alternate diagnosis that could account
for this acute presentation (terminal ileitis, Mickell’s diverticulitis, ovarian pathologies, ruptured
ectopic pregnancy, etc.). if another pathology is found, deal with it. If not, remove the appendix,
there is a very good chance that it harbors an early inflammation that will be shown by
histopathology.

There is absolutely nothing wrong about removing a normal appendix, as long as your decision
to proceed to surgery was based on genuine clinical findings.

_____________________________________

Q8 - Female post op with paralytic ileus. Labs showed hypokalemia.


What is the best investigation?

A - Urine osmolarity
B - Urine K
C - Stool K
D - ECG

Answer: D

The earliest ECG change associated with hypokalemia is a decrease in the T-wave amplitude. As
potassium levels decline further, ST-segment depression and T-wave inversions are seen.

_____________________________________

110
Q9 - 60-year-old man is seen for elective repair of ventral hernia. He has diabetes, hypertension,
and heart failure (He has all HF symptoms: ascites, pulmonary edema, lower limb edema).
What is the best management?

A - Open repair as scheduled


B - Lap repair as scheduled
C - Later if obstruction happened

Answer:
Elective surgeries should only be offered after appropriate optimization of medical condition.
A patient with decompensated heart failure should not be put to sleep for a procedure that can be
deferred till optimization takes place.

In addition, asymptotic or minimally symptomatic hernia can by managed by the “watchful


waiting” approach where surgery will only be offered in case complications develop.

_____________________________________

Q10 - 65-year-old, heavy smoker is coming for routine checkup.


What is the best screening test for him?

A - Osteoporosis
B - Colon cancer
C - AAA

Answer:
Colonoscopy is recommended for screening of colorectal cancer starting at the age of 50, if
normal, you repeat it ten years later

Screening for AAA by US is practiced in few countries only. It is not a standard thing to do.

Osteoporosis, I don’t know about its screening. You got to ask a rheumatologist about it.

If I get this question, I will go for colonoscopy.

_____________________________________

111
Q11 - DVT prophylaxis in a patient with femur fracture and high creatinine?

A - UFH
B - Warfarin
C - Enoxaparin

Answer: A
Low molecular weight heparin is execrated renally. It is not a good option in people with
impaired renal function.
Warfarin will need a minimum of three days or so to start working. It is good for long term
anticoagulation but for this particular indication.
Unfractionated Heparin is the right choice.

_____________________________________

Q12 - 50-year-old female presents to the clinic for evaluation of a long-standing abdominal
swelling, diagnosed as a hernia. She is scheduled to have the operation in the next two days. Her
history is significant for atrial fibrillation, on warfarin and her INR is 3.5.
Which of the following is appropriate management?

A - Vitamin K
B - FFP
C - Platelet transfusion
D - No intervention needed

Answer:
Patients on warfarin requiring an elective procedure should be admitted to the hospital few days
before the date of the planned procedure where they have their warfarin stopped and they will be
started on IV heparinization. This heparin “bridge” should be stopped 4-6 hours before surgery
and resumed sometime after surgery (the exact number of hours depends on the risk of
postoperative bleeding).
Warfarin will be re-introduced postoperatively along with IV Heparin till therapeutic level INR
is reached
Reversal of anticoagulation by transfusion of FFP and / or vit K is only needed in emergency
setting and should not be used in elective setting.
Platelet transfusion has no role

_____________________________________

112
Q13 - What is the cause of high fever in first 24 h in abdominal surgery patient?

A - Wound infection
B - Pneumonia
C - Atelectasis

Answer: C
Atelectasis, this is what they want to hear. This is a myth in surgery. There is absolutely no good
quality evidence that atelectasis causes fever not in the first day nor in any day after surgery. But
this is the answer you need to give.

_____________________________________

Q14 - DVT progressed to PE. What is the high dignostic test?

A - Spiral CT
B - ECHO

Answer:
This is a very short recall. It all depends on the clinical scenario and hemodynamic stability of
the patient. Here you find an overview of the topic.
Echocardiography can be useful for ruling-in a Pulmonary embolism but should not be the main
test for ruling out a pulmonary embolism. Echo findings include McConnell’s Sign, enlarged
RV, IVS flattening and the 60/60 sign.

Echo should not be used alone as a test to rule out pulmonary embolism” but due to the high
specificity, echo is adequate as “a rule-in test at the bedside in critical care settings”. Echo has no
role in hemodynamically stable patients with acute PE.

Echo is an effective way to monitor thrombolytic therapy. Within several hours after
thrombolytic therapy or suction embolectomy

Echo is also good in right ventricular dysfunction.


Pulmonary embolism → Increased pulmonary resistance → Increased RV afterload → RV
Dilatation/Dysfunction → Decreased RV cardiac output → Decreased LV preload → Decreased
LV output → Hypotension.

_____________________________________

113
Hernia

Q1 - Patient presents to ER with irreducible painful periumbilical bulge. He is vitally stable.


Examination is normal except for a tender peri-umbilical lump.
What is your management?

A - Observation
B - Urgent surgery
C - US abdomen

Answer: B
Irreducible PUH. Likely strangulated. Imaging doesn’t change management. Waiting is certainly
wrong.
The right management is urgent surgical exploration and repair.

_____________________________________

Q2 - Post laparoscopic hernia repair 7 days ago, comes because of pus coming out from the
wound. Examination reveals pus oozing from the wound.
What is the appropriate management?

A - Antibiotics
B - Drainage
C - Drainage and mesh removal
D - Open the wound and leave it open

Answer:
This really depends on whether mesh is exposed to this pocket of pus collection or not.
Anyways, the short and “safe” answer is C.
There is too much to be said about this topic, however.

_____________________________________

Q3 - 65-year-old female with asymptomatic femoral hernia.


What is the most appropriate management?

A - Observation
B - Open repair with mesh
C - Laparoscopic repair

Answer: B
As they are prone to obstruct and / or strangulate, femoral hernias should always be repaired.
Whether they are symptomatic or not.

_____________________________________

114
Q4 - Patient with umbilical swelling, abdominal distension, and exaggerated bowl sounds.
What is the appropriate management?

A - Repair with mesh


B - Observation
C - CT scan

Answer: A
Obstructed PUH. Repair with mesh

_____________________________________

Q5 - 24-year-old man in military, with bilateral inguinal hernia


What is the management?

A - Laparoscopic repair with mesh


B - Open repair with mesh
C - Open repair without mesh
D - Observation

Answer: A
For inguinal hernia, open repair is the standard of care, while laparoscopic repair remains in
option.
There are, however two solid and one relative indications for laparoscopic repair.
Solid indications are bilateral or recurrent hernias.
Relative indication, obesity

So, the answer is laparoscopic repair with mesh.

_____________________________________

115
Q6 - 5 days post hernia repair swelling, no redness, no signs of infection, mild tenderness.
What is the diagnosis?

A - Recurrence
B - Hematoma
C - Seroma
D - Infection

Answer:
5 days is too early for recurrence.

Infection is excluded as the scenario states “no signs of infection”

Both, hematoma, and seroma can present similarly. Even in real life practice, in many occasions
the only way to differentiate between the two entities is to aspirate the collection by a large pore
needle and see what comes out.
If you get these two options, go for seroma.

_____________________________________

Q7 - A female adult post open inguinal hernial repair presents with a reducible inguinal swelling
and positive cough impulse.
What is the most appropriate management?

A - US to rule out recurrence


B - CT to rule out recurrence
C - Open hernia repair
D - Laparoscopic hernia repair

Answer: D
Diagnosis of recurrence in this case is readily made by physical examination, hence, no further
imaging is required.
The only two SOLID indications for laparoscopic repair of inguinal hernia are: recurrent and
bilateral hernias.
Morbid obesity is a relative indication.

_____________________________________

116
Q8 - What type of mesh used in repair of ventral hernia?

A - Onlay
B - Infraolay
C - Subolay
D - Inolay

Answer:
I guess the question is not about mesh type as much as it is about mesh location in relation to the
fascia.

Onlay is above the fascia. Sublay is below it and inlay is within the defect itself.

Decision for location is multi-factorial. One needs more information to be able to take that kind
of decision
In all cases, one should strictly avoid plugging the defect with an inlay mesh

+ What about Vicryl doctor? Is it the preferred type to use?


+ Vicryl (polyglactin 910) mesh is a fully absorbable mesh that provides soft tissue coverage
only for couple of weeks before it des-integrates

It is certainly not a good option for definitive hernia repair. It’s only indication for use is in cases
of significant abdominal contamination like in cases of peritonitis, where there is loss of domain
and primary closure of the abdominal wall is not possible. Here you may use Vicryl to help in
closure, knowing FOR SURE that this patient will develop incisional hernia sometime soon, but
you accept that just for the sake of avoiding leaving the abdomen open for longer time.
Remember that putting a non-absorbable synthetic mesh in a contaminated abdomen is
contraindicated.
Having said that, Vicryl mesh is falling out of favor as we have much better ways to deal with
such a scenario of difficult closure of a hostile abdomen.
_____________________________________

Q9 - Patient with history of inguinal hernia surgery comes complaining that he feels like his
testes became smaller (no other symptoms).
What is the most likely cause?

A - Ligation of testicular artery


B - Ligation (or tension, not sure) of the external ring

Answer:
Incomplete question, some options are missing
The likely explanation is that extensive dissection took place, resulting in venous plexus
thrombosis.
If the question mentions: pain and swelling for some period postoperatively then atrophy, this is
ischemic orchitis secondary to venous thrombosis (most common)
_____________________________________

117
Q10 - 65-year-old female with a symptomatic femoral hernia.
What is the treatment?

A - Observation
B - Lap repair
C - Open repair with mesh

Answer: C
Femoral hernias are problematic. They tend to present with complications, obstruction, or
strangulation. This is largely due to the narrow nature of the femoral canal.
Symptomatic or not, these hernias should be repaired whenever they are diagnosed. Both, open
and laparoscopic repairs are equally good as long as the defect is augmented by a prosthesis
(mesh)
The answer here is: Open repair with mesh

_____________________________________

Q11 - Hernia palpated lateral and inferior to pubic tubercle?

A - Inguinal
B - Femoral

Answer: B
The femoral canal lies just below the inguinal ligament and lateral to the pubic tubercle.
Consequently, a femoral hernia will pass below and lateral to the pubic tubercle, whereas an
inguinal hernia will be seen above and medial to it.

_____________________________________

118
Q12 - 42-year-old mother presents with paraumbilical mass, On exam you find the mass tender,
irreducible, and negative cough impulse.
What is the appropriate next step?

A - Diagnostic Laparoscopy
B - Abdominal Us
C - Open repair
D - Biopsy

Answer: Looks like irreducible PUH. Probably strangulated (because of tenderness)


Cough impulse is typically absent in strangulated hernias.

There is no rule for biopsy as this is not a tumor.


US, though it will confirm diagnosis, it is not expected to change management. The patient needs
to be explored either ways.

The choice between laparoscopic and open repair is influenced by many factors (body habitus,
previous abdominal surgeries, hernia size, surgical expertise, type of mesh available, among
others).
Options here don’t include “laparoscopic repair “ which would be otherwise an excellent
approach. It rather says “diagnostic laparoscopy”.
So, the answer would be: open repair.

_____________________________________

Two related recalls:


Q13 - 70-year-old male with reducible inguinal hernia with mild pain?

A - Mesh repair
B - Open mesh repair
C - No treatment needed

Q14 - Elderly with abdominal pain and history of mild inguinal hernia. Examination shows
reducible inguinal hernia, no tenderness.

A - Urgent inguinal repair


B - Repair inguinal after 2 days
C - CT abdomen and pelvic.

Answer:
Minimally symptomatic hernias need no treatment.
So for both recalls, watchful waiting.
_____________________________________

119
Breast Surgery

Q1 - 35-year-old female, her father developed colon cancer when he was 55 and her mother
developed breast cancer when she was 43.
Asking about screening?

A - She should do mammogram annually


B - Start mammogram at 40
C - Start mammogram at 40 and colonoscopy at 55
D - Colonoscopy

Answer: A

_____________________________________

Q2 - The following are appropriate methods for the treatment of inflammatory processes in the
breast EXCEPT?

A - Sporadic lactational mastitis treated with antibiotics and continued nursing.


B - Recurrent periareolar abscess with fistula treated by distal mammary duct excision.
C - Breast abscess treated by incision and drainage.
D - Breast abscess treated with antibiotics.
E - Thrombophlebitis of the superficial veins treated by reassurance of the patient and follow up
examination only.

Answer: D

_____________________________________

Q3 - 53-year-old woman had a mammogram now and was normal.


When should she do it again?

A - 6m
B - 1y
C - 2y
D - 3y

Answer: B

_____________________________________

120
Q4 - A female patient with breast ulcers and oozing from the nipple. Mammogram, no masses.
What to do?

A - Referral to dermatologist
B - Nipple biopsy
C - US at 6 months
D - Reassurance

Answer: B
Nipple biopsy to rule out Paget’s disease.
Paget's disease of the nipple is a rare form of breast cancer in which cancer cells collect in or
around the nipple. The cancer usually affects the ducts of the nipple first, then spreads to the
nipple surface and the areola. The nipple and areola often become scaly, red, itchy, and irritated.
Paget's disease of the nipple accounts for less than 5% of all breast cancer cases.

_____________________________________

Q5 - Female after resection of fibroadenoma they found single focus of DCIS.


What is the best management?

A ⁃ Chemo
B ⁃ Radiotherapy
C ⁃ Mastectomy

Answer:
Wrong question
There should be an option of “resection to negative margin”
Also, mammographic findings and lump / breast size are important information.
_____________________________________

Q6 - Post-partum women presents with breast pain and difficulty breastfeeding. On exam there is
no mass or fluctuation.
What is the most appropriate next step in management?

A - Give antibiotics that covers staph aureus and do US to rule out abscess
B - Discontinue breast feeding
C - Discharge with good analgesia and supportive bra

Answer:
In absence of genuine indication like abscess or mastitis, starting antibiotics wound be very
inappropriate.
Discontinuing breast feeding should be strongly discouraged.
Would go for C.
_____________________________________

121
Q7 - A female patient comes with a breast lump.
Which one of the following doesn’t need follow up?

A - Cystic lesion with serous fluid that does not refill again.
B - Blood on aspiration
C - Solid lesion
D - Fibrocystic changes on histologic examination

Answer: A

_____________________________________

Q8 - A female patient comes with white breast discharge and high prolactin.
What radiologic study will you do?

A - Pelvic MRI
B - Chest MRI
C - Brain MRI

Answer: C

_____________________________________

Q9 - Breast mass behind the nipple. On US, hypoechoic lesion.


What is the next step?

A - Core biopsy
B - FNA

Answer: A
FNA can’t differentiate in-situ from invasive carcinoma. Tru-cut (core needle) biopsy is the right
modality.

_____________________________________

122
Q10 - 47-year-old woman, single, with positive family history of breast cancer, underwent
routine mammography which showed bilateral increased density and glandular pattern. Core
needle biopsy showed atypical ductal hyperplasia.
What’s the most appropriate management?

A - Wide local excision.


B - Simple mastectomy
C - Observation

Answer: A
Atypical ductal hyperplasia (ADH) is a pathologic finding in breast tissue.
Atypical ductal hyperplasia is usually identified incidentally on specimens obtained by needle
biopsy prompted by abnormal findings on mammography. Atypical ductal hyperplasia correlates
with an increased risk of breast cancer and therefore classified as a "high risk" lesion but is not a
"precursor" lesion - the distinction being the breast cancer associated with ADH can occur
anywhere in the breasts and not only in the area of the ADH.

_____________________________________

Q11 - After fibroadenoma excision, histopathology report showed lobular carcinoma in situ with
free margins.
What is the most appropriate management?

A - Mastectomy
B - Chemotherapy
C - Radiotherapy

Answer:
LCIS is not a disease. It doesn’t require treatment on its own. It is rather a marker for increased
risk of breast cancer. The risk applies to both breasts.

D- Close surveillance

_____________________________________

123
Q12- 32-year-old lady presents with bloody nipple discharge. Examination is normal. What is
the next investigation?

A - Mammogram
B - US
C - MRI

Answer:
The choice between US and Mammo.
US < 30
Mammo > 35

30 - 35 no consensus. It really depends on radiologist and thickness of breast parenchyma


(multiparous women will have more fatty replacement of their breast parenchyma that makes
mammo a perfect choice)

_____________________________________

Q13 - Breast rash affecting the areola. Has been on topical steroids with no improvement. No
nipple discharges?

A - Antibiotic
B - Surgery
C - Mammography

Answer: C
Paget disease of the breast. Needs imaging and biopsy

_____________________________________

Q14 - What is the most significant risk factor for breast cancer?

A - Age
B - Early menarche
C - Late puberty
D - Delaying childbirth until after 30

Answer: A
Most significant risk factor is female gender.
In this question it is age.

_____________________________________

124
Q15 - Female presents with progressively enlarged breast mass.
imaging showed: cystosarcoma phyllodes
What is the most appropriate management?

A - Radiotherapy
B - Neoadjuvant chemotherapy
C - Modified radical mastectomy
D - Simple mastectomy

Answer: D

Phyllodes tumor, also known as cystosarcoma phyllodes, is a rare fibroepithelial tumor of the
breast which has some resemblance to a fibroadenoma. It is typically a large, fast-growing mass
that forms from the periductal stroma of the breast.
It accounts for less than 0.3-1% of all breast neoplasms. It is predominantly a tumor of adult
women, with very few examples reported in adolescents. The occurrence is most common
between the ages of 40 and 60, before the menopause (peak incidence ~45 years). This is about
15 years older than the typical age of patients with fibroadenoma.
Patients typically present with a painless, rapid growing breast mass for which imaging is
requested.
A phyllodes tumor may be considered benign, borderline, or malignant depending on histologic
features including stromal cellularity, infiltration at the tumor edge, and mitotic activity.
FNA is inaccurate, and even core biopsy has moderate sensitivity due to tumor heterogeneity
causing inadequate sampling.
It is a locally invasive tumor. Treatment is usually with surgical excision. Large tumors may
require simple mastectomy. Both benign and malignant phyllodes tumors tend to recur if not
widely excised. Malignant degeneration is seen in 5-25%
After wide local excision, there is relatively frequent local recurrence (up to 25%) and up to 10%
can metastasize. The mode of metastases in such cases is by hematogenous route.

_____________________________________

Q16 - Screening for 35-year-old woman with family history of ovarian and breast malignancy?

A - BRCA
B - Mammo
C - US

Answer:
Need more details
BRCA testing indications must be met.
What is the age of youngest affected family member?!!

With the available data, I will go for B.

_____________________________________

125
126
Q17 - 25-year-old female with a painful breast mass 2 x 2 cm, described as oval shape.
What is the most likely diagnosis?

A - Duct ectasia
B - Breast cyst
C - Breast cancer

Answer: B
_____________________________________

Q18 - Pregnant woman comes to the clinic concerned about development of non-tender lumps
confined to her areola.
What is the diagnosis?

A - Lactiferous duct
B - Mondor's disease
C - Montgomery follicles

Answer: C
Montgomery’s tubercles are sebaceous glands that appear as small bumps around the dark area
of the nipple. 30 and 50 percent of pregnant women notice Montgomery’s tubercles.

_____________________________________

Q19 - Excuse me doctor I have a question regarding duct ectasia


Is it diagnosed by US/mammo only or I must do core needle biopsy to diagnose and do duct
excision; I couldn’t find a source explaining this.

Answer:
Ductectasia is dilatation of milk ducts with wall thickening.
Like all benign breast diseases, you manage it according to symptoms and radiology findings

Usually, it is described based on US


If it is found incidentally in US and it was coded as BIRAD 2 then nothing extra is needed.
Patient should go for her usual screening program
If BIRAD 3 then you need follow up radiology
If the duct looked suspicious and coded as BIRAD 4 we discuss with the radiologist about either
taking biopsy or microdochectomy.
If Patient presents with nipple discharge you need MRI followed by microdechotomy.
If presents with a mass you manage it like a breast mass, decision for further surgery depends on
biopsy finding / radiology pathology correlation and mass size.
It should not present as skin or nipple changes.

_____________________________________

127
Q20 - 35-year-old female presents to outpatient clinic with a hard 3 x 4 cm mass in the upper
outer segment of the left breast. Examination showed nipple retraction but no axillary lymph
node involvement. Mammography shows an ill-defined mass that's suspicious.
How would you biopsy this?

A - True-cut
B - Excisional
C - Incisional
D - FNA cytology

Answer: A
True-cut biopsy is the preferred way of obtaining diagnosis in breast cancer.
FNA can’t differentiate between invasive and in-situ lesions, and you can’t make any therapeutic
decision based on its result.
Incisional biopsy will violate tumor planes and risk disseminating malignant cells.
Excisional biopsy, though good for some benign lesions, is totally contraindicated in cases of
suspected breast cancer

_____________________________________

Q21 - 55-year-old female comes with bilateral breast pain bilateral green discharge from
multiple ducts. Imaging: Multiple dilated ducts, not suspicious.
What to do?

A - Core needle biopsy


B - Interval follow up imaging
C - Galactogram
D - Surgery

Answer:
They must specify whether the discharge is spontaneous or not.

If nonspontaneous then advise to stop milking the breast and reassess

If spontaneous (rare scenario to be bilateral) get an MRI and if the MRI is negative for a mass,
discuss with the patient the option of bilateral total
dochectomy, especially if the discharge is bothering her and staining her under garment.

_____________________________________

128
Q22 - Female underwent routine mammography which showed bilateral increased density and
glandular pattern. Core needle biopsy showed atypical ductal hyperplasia.
What is the appropriate management?

A - Wide surgical excision.


B - Simple mastectomy

Answer: A
ADH in core needs excision. The right answer is wide local excision

_____________________________________

Q23 - Intraductal hyperplasia in patient with family history of breast cancer.


What’s the appropriate management?

A - Simple mastectomy
B - Wide excision
C - Radical mastectomy
D - Preventive chemo

Answer: B
ADH in core needs excision. The right answer is wide local excision

_____________________________________

Q24 - Lactating mother red, tender, erythematous breast, already started on floxacillin. US shows
2 x 3 cm cystic fluctuating lesion.
What is the best management?

A - Observe
B - Repeated aspiration
C - Incision and drainage

Answer: B
Breast abscess. Small ones can be needle aspirated. Big ones will need to be incised.

_____________________________________

129
Q25 - Woman in her 30s, with breast mass 15 x15 cm, after resection, histopathology showed
cystosarcoma phyllode.
What is the best management?

A - Simple mastectomy
B - Modified radical mastectomy
C - Radical mastectomy

Answer: A

_____________________________________

Q26 - Female with breast mass, first step in management?

A - US both breasts
B - FNA

Answer: A
Incomplete question. Age is important. Imaging is certainly before biopsy.
Among these two options provided, go for A.

_____________________________________

130
Thyroid Surgery

Q1 - A young lady pregnant at 28 weeks, known case of hyperthyroidism on 15 mg carbimazole


came with symptoms of hyperthyroidism. Labs: low TSH high T4.
What's the most appropriate management?

A - Switch to PTU
B - Increase carbimazole dose to 20 mg
C - Referral for thyroidectomy
D - RAI

Answer: B

_____________________________________

Q2 - What’s the most common site of ectopic parathyroid gland?

A - Para esophageal
B - Thymus

Answer:
Ectopic inferior parathyroids are most frequently found in the anterior mediastinum, in
association with the thymus or the thyroid gland, while the most common position for ectopic
superior parathyroids is the tracheoesophageal groove and retroesophageal region.

_____________________________________

Q3 - Female patient had left inferior parathyroidectomy then she developed fatigue and elevated
parathyroid hormone
What is the cause?

A - New adenoma
B - Missed adenoma
C - Parathyroid hyperplasia
D - Parathyroid caner

Answer:
Incomplete question.
We don’t know about pre-operative work up, especially US.
In this format, both B and C are possible. Hope you get a better recall

_____________________________________

131
Q4 - 32-year-old pregnant lady in third trimester presents with palpitations, sweating, heat
intolerance and restlessness. She has a large goiter but no dyspnea or difficulty swallowing. She
also has changes in both orbits that became more prominent.
ECG, sinus tachycardia.
CBC -LFT - RFT within normal.
TSH 0.1 (low), T4 10 (high).
Thyroid Scan showed diffuse homogenous uptake.
Anti-Thyroid Stimulating Antibody is pending.
What is your next step for management?

A - Thyroidectomy
B - Radio-active Iodine Therapy
C - Methimazole
D - Propylthiouracil.

Answer: C
This is a clinical picture of hyperthyroidism. Biochemically, her TSH is only mildly suppressed
(0.1) and T4 is actually normal, which makes her “subclinical” hyperthyroidism. What is missing
in this question is T3 level. You will have to remember that there is a separate clinical entity
called “Isolated T3 toxicosis”. This piece of information- if provided- can change the whole
scenario.
Radioactive iodine is contraindicated in pregnancy.
Propylthiouracil can only be given in first trimester.
Methimazole is the way to go (in case T3 was high). But if T3 was normal you should go for
something like beta blockers to control her symptoms.
Now, back to this scenario, with this much information provided, the answer is Methimazole.

By the way. this is a pure endocrinology question. There is nothing surgical about it. But I
answered it because it is a very interesting question

_____________________________________

Q5 - Picture of hyperthyroidism with US showing right solid nodule 2 cm and thyroid scan
showing picture of toxic multinodular goiter.
What is the most appropriate management?

A - Subtotal thyroidectomy
B - Iodine Radio-ablation
C - Right thyroidectomy
D - Near complete thyroidectomy

Answer: D

_____________________________________

132
Q6 - Patient with symptoms of hyperthyroidism and exophthalmos. On thyroid scan there is an
increase uptake, and it was suggestive of nodular goiter in the right side (0.5cm).
What’s the management?

A - Right hemi
B - Near total
C - Radioactive ablation

Answer: B
Eye symptoms are usually seen with Graves rather than toxic nodules

_____________________________________

Q7 - Long case of medullary thyroid cancer (diagnosis given).


What is the appropriate management?

A - Subtotal thyroidectomy
B - Total thyroidectomy
C - Hemithyroidectomy

Answer: B

_____________________________________

Q8 - A female patient hours after left thyroid lobectomy complains of severe shortness of breath
and points to her neck.
What is next step in management?

A - Take her back to OR


B - Bedside tracheostomy
C - Nasal cannula

Answer:
This is a wound hematoma causing extrinsic compression on upper airway.
Treatment is BEDSIDE evacuation of hematoma by opening the wound THEN taking the patient
to OR to achieve hemostasis.
So, the answer must be “D” Bedside evacuation of hematoma by opening the wound

_____________________________________

133
Q9 - 45-year-old woman complains to her primary care physician of nervousness, sweating,
tremulousness, and weight loss. The thyroid scan reveals a single focus of increased isotope
uptake.
Which of the following is the most likely diagnosis?

A - Hypersecreting adenoma
B - Graves’ disease
C - Multinodular goiter
D - Papillary carcinoma of thyroid

Answer: A
Hypersecreting adenoma is a term used to describe parathyroid gland pathologies and not thyroid
gland ones.
The answer is toxic thyroid nodule.

_____________________________________

Q10 - Patient with left thyroid lump. Thyroid ultrasound shows 1.5 x 2 cm mass. FNA,
suspicious follicular neoplasm.
What is the management?

A - Left hemithyroidectomy
B - Repeat FNA
C - Antihyperthyroid drugs
D - Radioactive ablation

Answer: A
Options C and D would be considered for a hypersecreting nodule.
B is not needed as FNA which was done is adequate.
Go for surgery. Left hemi-thyroidectomy.
_____________________________________

134
Q11 - Normal thyroid. Cervical lymph node biopsy showed normal follicular thyroid cell.
What is the diagnosis?

A - Ectopic thyroid
B - Follicular carcinoma
C - Lymphoma
D - Aberrant

Answer: D
The thyroid gland develops from 2 embryonic origins: one medial endodermal, which produces
most of the thyroid parenchyma, and 2 lateral (branchial bodies), formed by the 4th pharyngeal
pouch that give rise to C cells.
In its embryonic development, the thyroid descends through the thyroglossal duct from the base
of the tongue towards the anterior tracheal wall. Incorrect descent can lead to an ectopic thyroid.
This anomaly appears in 7% of the population. The most frequent locations are in the area of the
descent on the midline and are classified as lingual, sublingual, thyroglossal and
intralaryngotracheal. The lingual location is the most frequent and represents around 90% of
cases.3 Other less frequent locations are: cervical lymphatic nodules, pericardium or cervical
oesophagus.
Traditionally, the thyroid tissue located in the lateral cervical compartments was known as lateral
aberrant thyroid tissue and was believed to be an embryological variation. Currently, this is not
clear, and many authors defend its origin as metastatic deposits of well-differentiated thyroid
carcinoma.
_____________________________________

Q12 - What is the most common indication for surgery in Grave’s disease?

A - No response to medications
B - Eye symptoms

Answer: B
As patients with eye signs are refractory to radioactive iodine.

_____________________________________

Q13 - Thyroid enlargement (hot nodule). FNA normal.


What is the next step?

A - Thyroid scan
B - Repeat FNA
C - Start antithyroid therapy

Answer: C
_____________________________________

135
Q14 - Young male with 4-month history of neck pain. TFT is normal. US: 9 x 7 mm solid nodule
with no cervical lymph node enlargement.
What is the management?

A - Follow up imaging
B - FNA

Answer: A
As per guidelines of American Thyroid Association (ATA), FNA is indicated for nodules larger
than 1 cm.
_____________________________________

Q15 - A female had left neck mass that moves with swallowing. US showed 2.5 cm solid thyroid
mass. (They didn’t provide the TFT).
What will you do next?

A - Core needle biopsy


B - FNA
C - Thyroid scan
D - Follow up

Answer is: B
_____________________________________

Q16 - Strongest indication of surgery in graves?

A - Pediatric
B - Presence of eye symptoms.
C - Presence of anti TSH.
D - Failed antithyroid meds.

Q17 - Most common indication of surgery in graves?


A - Pediatric
B - Presence of eye symptoms.
C - Presence of anti TSH.
D - Failed antithyroid meds.

Answer: B
Eye symptoms, as patient with eye symptoms are refractory to medical treatment.

_____________________________________

136
Q17 - Patient with neck pain. Thyroid function test normal. US, 4 mm solidarity nodule.
What is the most appropriate management?

A - FNA
B - No need for FNA
C - Thyroid scan
D - Thyroid lobectomy

Answer: B
FNA for thyroid lesions smaller than 1 cm has a very low yield and is not recommended.
Thyroid scan is done for hypersecreting lesions.
Thyroid lobectomy is not indicated as there is no diagnosis.

_____________________________________

Q18 - Female known to have Hashimoto’s thyroiditis, presents with rapidly growing nodule,
over the past 4 month.
What is the most likely diagnosis?

A - Papillary
B - Lymphoma

Answer: B

_____________________________________

Q19 - Hot thyroid nodule 3 x 3 cm was removed by thyroid lobectomy. Histopathology showed
8mm papillary carcinoma.
What is the most appropriate management?

A - Completion thyroidectomy
B - Follow up 3 months
C - RAI
D - Chemotherapy

Answer: B
Guidelines for management of such patient have changed.
As opposed to previous recommendations advocating completion thyroidectomy for all PTC,
current guidelines encourage close surveillance as risk of recurrence is not high enough to justify
operating on every patient.

_____________________________________

137
Q20 - 39-year-old male presents to the clinic with a 3-week history of painless neck mass. On
examination, the mass is located at level 2. It is hard and not tender, measures 3 x 5 cm.
What is the best next step in management?

A - CT with contrast
B - US

Answer: A
Likely an enlarged lymph node. CT is the modality of choice.

_____________________________________

138
Q21 - Patient with medullary thyroid cancer what is the mangment?

A - Total thyroidectomy
B - Subtotal thyroidectomy
C - Hemithyroidectomy

Answer: A
Medullary thyroid carcinoma (MTC) is a rare neuroendocrine tumor derived from the thyroid C
cells producing calcitonin. MTC accounts for 0.6% of all thyroid cancers.

All patients with a preoperative diagnosis of MTC should undergo detailed neck ultrasonography
(US) and measurement of serum calcitonin and carcinoembryonic antigen (CEA). Basal serum
calcitonin concentrations usually correlate with tumor burden but also reflect tumor
differentiation in MTC. Thus, total thyroidectomy and dissection of cervical lymph node (LN)
compartments, depending on serum calcitonin levels and neck US findings, is standard treatment
for patients with MTC. In addition, establishing whether the patient has germline rearranged
during transfection (RET) mutation or sporadic disease is crucial because patients with MEN
type 2 can have pheochromocytoma or primary hyperparathyroidism, or both. If the patient has
pheochromocytoma, adrenalectomy should be prioritized before thyroidectomy.
For patients with MTC confined to the neck and no evidence of involved cervical LN on
preoperative imaging studies and no evidence of distant metastases, total thyroidectomy with LN
dissection in the central compartment is recommended as the preferred initial treatment.
However, prophylactic central neck LN dissection is not required in patients with small
intrathyroidal MTCs with a preoperative calcitonin level <20 pg/mL, because there is virtually
no risk of LN metastases when the preoperative serum basal calcitonin level is less than 20
pg/mL (normal reference value, <10) . An important issue is whether lateral LN dissections
should be performed in the absence of clinically or ultrasonographically detectable LN
metastases as part of the primary surgery. In patients with MTC and no evidence of neck
metastases on US, and no distant metastases, dissection of LN in the lateral compartments (levels
II to V) may be considered based on serum calcitonin levels according to the revised American
Thyroid Association (ATA) guidelines. For patients with MTC confined to the neck and cervical
LNs, total thyroidectomy, dissection of the central LN compartment, and dissection of the
involved lateral neck compartments should be performed. When preoperative imaging shows
positive results in the ipsilateral neck compartment but negative results in the contralateral neck
compartment, contralateral neck dissection should be considered if the basal serum calcitonin
level is greater than 200 pg/mL.

_____________________________________

139
Q22 - Right sided hot nodular goiter 2 x3 cm, with exophthalmos (eye protrusion).
What is the most appropriate management?

A - Near total thyroidectomy


B - Right hemithyroidectomy
C - Radioactive ablation

Answer: A

_____________________________________

Q23 - Patient with neck pain. Thyroid function tests normal. US, solitary
nodule, 4 mm in size.
What is the next appropriate step?

A - FNA
B - Thyroid Scan
C - Thyroid lobectomy
D - No need for FNA

Answer: D
Pain can’t be attributed to this tiny nodule.
This is an incidental finding. No indication for any further interventions, including FNA.

_____________________________________

140
Pediatric Surgery
Q1 - 5-year-old girl presents to the Emergency Room with history of severe abdominal pain,
which is central and colicky, and repeated vomiting. Pain is getting worse by movement. On
examination, there is generalized guarding, with more tenderness toward the right iliac fossa,
Tachycardic and febrile (T=38). WBC 15 and ESR is 12.
Which of the following is right regarding this condition?

A - Occurs in 20% individuals


B - It's uncommon below 3 years old
C - Sausage shaped mass often palpable
D - Passage of redcurrant jelly stool confirms the diagnosis

Answer: B
Acute appendicitis, likely perforated.
Acute appendicitis is common surgical emergency among children. Overall, 1–8% of children
presenting with abdominal pain have acute appendicitis. However, Appendicitis is uncommon in
pre-school Children
_____________________________________

Q2 - 3-year-old child with painless blood with stool. When doctor examined him he said this
pathology can recur.
What is the diagnosis?

A - Anal fissure
B - Hemorrhoids
C - Intussusception

Answer: B

_____________________________________

Q3 - Patient with undescended testis not palpable.


What is the best modality?

A - MRI
B - Testosterone level sampling
C - Laparoscopy
D - PET

Answer: C
Laparoscopy is the way to go. It is both diagnostic and therapeutic, in case testicles are found.

_____________________________________

141
Q4 - What is the most common site of intussusception in pediatrics?

A - Ileocolic
B - Ileoileal
C - Ileocecal

Answer: C
_____________________________________

Q5 - Child comes with vague abdominal pain for 2 weeks. He has bruises on the abdomen. He
tells you he fell from bicycle 18 days ago.
What is your next action?

A - Abdominal US
B - Abdominal CT

Answer: B
Traumatic acute pancreatitis.
Although US might be a good start in pediatrics, it has very limited ability to visualize the
pancreas due to overlying bowel gas.
Go for CT
_____________________________________

Q6 - Infant with painful reducible umbilical hernia?

A - Observation (or conservative)


B - Surgery
C - Wait until 6 months of age

Answer: A
Most umbilical hernias will close as the child grows. If hernia persists till school age, surgical
repair is indicated. It will be open repair without mesh.

_____________________________________

Q7 - Pediatric umbilical mass and gets bigger when baby cries, treatment?

A - Laparoscopic repair with mesh


B - Open repair
C – Herniotomy

Answer:
Refer to previous question

_____________________________________

142
Q8 - 5-month-old baby presents with bilateral groin swelling that has been present since birth.
On examination, there are small bilateral inguino-scrotal swellings, which become prominent
when the child cries. His abdomen is soft and non-tender.
Which of the following is the most appropriate management?

A - Herniotomy
B - Lichtenstein repair
C - Laparoscopic mesh repair
D - Biological mesh placement

Answer: A
Pediatric hernia repair never with mesh. Just reduce the contents and amputate hernia sac.

_____________________________________

Q9 - A 12-year-old boy received blunt trauma on his abdomen and later presents with
generalized abdominal pain. Imaging of the spleen showed a 7 mm hematoma and 4 cm tear
(grade 3 injury).
What is the management?

A - Splenectomy
B - Spleen preserving surgery
C - Conservative

Answer: C
Irrespective of these findings on CT, as long as the patient’s hemodynamics are maintained, you
pursue a non-operative approach

_____________________________________

Q10 - Child with grade 3 spleen injury after blunt abdominal trauma. He is stable.
What is the best management?

A - Close observation
B - Spleen preserving surgery
C - Splenectomy

Answer: A
Conservative treatment
Management of splenic injuries is based on hemodynamic stability.
A stable patient should be conserved.

_____________________________________

143
Urology

Q1 - 82-year-old man presents with very painful micturition and week stream.
What is the most appropriate management?

A - Antibiotics for UTI


B - Foley catheter and antibiotics
C - Cystoscopy and TURP

Answer: B
Looks like BPH with UTI. Needs antibiotics and Foley for drainage to facilitate control of
infection.

_____________________________________

Q2 - 67-year-old male complains of painless hematuria.


What is the diagnostic test?

A - Cystoscopy
B - CT abdomen
C - IV pyelogram
D - US

Answer: A
It is bladder cancer until proven otherwise

_____________________________________

Q3 - What is the most common part of urethra affected by trauma in male?

A - Bulber
B - Penile
C - Membranous
D - Prostate

Answer: A
Urethral injuries can be classified based on location as either anterior or posterior. Anterior
urethral injuries are often the result of blunt or penetrating trauma with the bulbar urethra the
most common location affected.

_____________________________________

144
Q4 - Severe flank pain with hematuria and painful micturation?

A - CT with oral and iv contrast


B - CT without contrast
C - Xray
D - US

Answer: B
Contrast gets execrated by kidneys. This will obscure renal, ureteric, or bladder stones. Hence,
go for CT without contrast.

_____________________________________

Q5 - Patient with sudden loin pain and vomiting. She also has hematuria (from labs). She is
rolling around from pain.
What is the likely diagnosis?

A - Ureteric stone
B - Acute pyelonephritis
C - Acute cystitis

Answer: A

_____________________________________

145
Bad Recall

Here is a collection of some bad recalls. Some of them are worse than others. The reason I
created a special section for them is for you to learn “Critical Appraisal Thinking “, which
is the analysis of facts to form a judgment, whenever you approach an MCQ.
If the question doesn’t look right, then it is not right. Just move on and go read about the
topic. When the question appears in the exam, it will certainly be written in a way that you
can answer.

Q1 - Patient had cholecystectomy and cystic duct was injured.


How would you repair it?

A - Hepato jejunostomy
B - Hepato deudeontomy
C - Choldejejuonestomy
D - Choldochodeunostomy

Answer: Wrong question


In cholecystectomy we “cut“ cystic duct. The question must be about another duct (CBD, CHD,
RHD, LHD)!!!

_____________________________________

Q2 - Post cholecystectomy with abdominal pain and distribution. US confirms


presence of ascites.
What to do?

A - Percutaneous aspiration
B - Laparotomy
C - ERCP

Answer: Distribution?!!!
Ascites?!!!
I bet you mean “collection”
Percutaneous drainage

_____________________________________

146
Q3 - Patient post foot crush injury with a heavy object comes with excruciating pain and pale
cold limb.
Best next step?

A - X-ray
B - CT
C - Compartment pressure

Answer: Question is wrong. It can’t be “foot crush injury”. It should be lower leg.
This is compartment syndrome. It needs to be confirmed by measuring compartmental pressure

_____________________________________

Q4 - Patient with right thyroid nodule. It is hot and the rest of gland is cold
What is the treatment?

A - Right lobectomy
B - Total thyroidectomy
C - Thyrotoxic drugs

Answer: You need to know TFT


If the question comes in this format, go for right hemithyroidectomy

_____________________________________

Q5 - 38-year-old woman presents with a painful breast mass about 4 cm in the upper outer
quadrant. It increases in size with the menstrual period. Examination showed tender nodularity
of both breasts.
What is the management?

A - Hormonal treatment with oral contraceptive pills


B - Hormonal treatment with danazol
C - Lumpectomy
D - Observation for 6 months

Answer: All four options are wrong.


You need imaging (Mammo +/- targeted US)

_____________________________________

147
Q6 - Patient presents with breast abscess was on antibiotics but no improvement
What to do?

A - Change the antibiotics


B - Stop feeding
C - Encourage breast feeding
D - Do US and call surgery for review

Answer: Diagnosis was made. Abscess should be drained.

_____________________________________

Q7 - Crhon’s with a painful perianal mass. There is discharge when touch.


What is the management?

A - Pelvic MRI
B - Fistulotomy
C - Increase dose of infliximab

Answer: This is a perianal abscess.


Treatment is incision and drainage

_____________________________________

Q8 - Patient was going for weight reduction surgery. On endoscopy found to have grade III
hiatal hernia.
What is the bariatric procedure of choice?

A - Sleeve gastrectomy
B - Roux-en-Y gastric bypass
C - Balloon dilation

Answer: There is nothing called grade three hiatal hernia. I’m sure you are referring to hiatal
hernia with grade three esophagitis.
You go for RYGP

_____________________________________

148
Q9 - Appendicular abscess 15 x17cm.
What is the management?

A - Antibiotics
B - Surgery

No drainage in the options

Answer:
There must be percutaneous drainage.
This is the standard of care.
_____________________________________

Q10 - Child with history of jelly like stool the patient was stable and not obstructed next
management?

A - NGT and NPO


B - Warm enema under us
C - Laparotomy excision
D - Laparotomy reduction

Answer:
This is a case of Intussusception

The question is asking about the “next step”, now does that ‘next’ mean “initial” - NGT and
NPO - or “definitive” – enema -?!!

Looking at the other options, however, it is difficult to imagine that someone will put laparotomy
twice in a question that talks about “initial” management. I would go for B.

_____________________________________

Q11 - 25-year-old woman, is asking about best timing for breast self-examination

A - 6-7 day after cycle


B - 5 day before
C - 7-10 day after
D - 14-16 day after
E - After 2 days

Answer: As per guidelines, breast self-exam as a screening modality is no longer recommended.


Doctors advise their patients to do it, but they must explain its limitations.
Moreover, the question says “after” period. We don’t know is it after the start or the end of
period.
“A” might be the closest answer.
_____________________________________

149
Q12 - Female patient presents with right upper quadrant pain and fever. Lab tests show increased
WBCs, increased ALP, increased Direct bilirubin. US Show: fluid around gallbladder, multiple
stones.
What is the most appropriate management?

A - Lap chole
B - Laparotomy
C - Precautions guide drainage
D - Antibiotic

Answer: RUQ pain + fever + jaundice = Charcot triad.


The other two pieces of information given in the question:
- High white count (it goes with the diagnosis of ascending cholangitis).

- Fluid around gallbladder. If this means “pericholecystic fluids” then this is a picture of acute
cholecystitis. If they mean fluid collection, then this is a perforated gallbladder.

So, let’s agree that there are two diagnoses in this scenario, which is ok, patients may come with
ascending cholangitis and acute cholecystitis (whether perforated or not) concomitantly.

Now for the options. Laparotomy certainly not. Lap chole, although will be needed at some point
but it is not an option now as it doesn’t address the issue of cholangitis. Likewise, percutaneous
drainage is noncontributory in management of cholangitis and has absolutely no role if we are
talking about pericholecystic fluids.

We are left with antibiotics. Although it is a very integral part in management of both
pathologies, the key to control sepsis in ascending cholangitis is biliary decompression via ERCP
which doesn’t appear among the options.

_____________________________________

150
Q13 - A woman presents 5 days (I think) after C-section complaining of abdominal pain and
tenderness, I think there was distention too,
On imaging they found that she has small bowel fistula
What are you going to do?

A - Colostomy
B - Resection and anastomosis
C - Soft diet only
D - NPO and give pancreatic enzyme inhibitors

Answer: Fistula (by definition) takes 6 weeks to develop. This is actually the time needed for
fibrous tissues formation

The question is probably about small bowel injury complicating the recent CS.

The answer would be surgical exploration (open or laparoscopic), peritoneal washout followed
by resection and anastomosis of the involved segment.

_____________________________________

Q14 - Patient with bleeding per rectum and fluctuated PSA level.
What is the management?

A - Antibiotics
B - OPD referral
C - Bedside aspiration

Answer:
PSA is prostate specific antigen. It’s a tumor marker for prostate cancer. PSA is not related to per
rectum bleeding.

_____________________________________

151
Q15 - Patient four months after gastric bypass comes to ER with 12 abdomen tenderness.
Next?

A - Endoscope
B - Barium

No CT in the options

Answer:
Incomplete scenario…
One would want to have more information about clinical presentation.
Possibilities range from marginal ulcers to internal hernia to gallbladder pathology to late leak,
among others.
_____________________________________

Q16 - 29-year-old lady presents with central abdominal pain for three days, nausea and vomiting
for one day. Her surgical history includes laparoscopic sleeve gastrectomy 6 years ago. Physical
examination reveals dehydrated with distended abdomen and exaggerated bowel sounds.
What is the likely diagnosis?

A - Incisional hernia
B - Internal hernia
C - Intussusception
D - Adhesion

Answer:
This lady is clearly obstructed. The cause of obstruction, however, can’t be reliably concluded
from the scenario.

Internal hernia and intussusception are two problems peculiar to Roux-en-Y Gastric Bypass.
They can’t happen in sleeve.

Incisional hernia can complicate any abdominal surgery, the scenario, however, didn’t tell us
about any “bulge” over the scars of her previous surgery.

What are we left with, adhesion...


Although adhesive bowel obstruction can “theoretically” complicate any abdominal surgery, it is
unheard of after sleeve gastrectomy.

If one changes the procedure on the question to RYGP instead of sleeve, then the answer would
be internal hernia.

_____________________________________

152
Q17 - Patient after abdominal surgery develops shock (fever, tachycardia, hypotensive).
What is the initial step in management?

A - Exploration
B - FAST
C - X-ray

Answer: Incomplete scenario.

We don’t know what kind of surgery patient underwent. We don’t know the timeframe of
development of this complication. Complications that develop in day one postoperatively are
entirely different from those that develop on day 7 for example.
Having said that, the scenario in hand describes a picture of septic shock. Possibilities are,
collection, anastomotic leak (if there was an anastomosis), missed iatrogenic bowel injury, and
so on.
Such patient should be resuscitated with IVF and broad-spectrum antibiotics started empirically
while an active search is undertaken to find the underlying cause of this sudden deterioration.
X-Ray has no role here because even if it shows air under diaphragm, this has no significance as
this patient is expected to show air for quite some time after abdominal surgery

FAST has no role in non-trauma patients

Exploration - though likely to be needed at some point - can’t be considered as an “initial” step.
It is rather a definitive treatment

So, in a question that asks about “initial step”, go for something like IVF or broad-spectrum
antibiotics.

_____________________________________

153
Q18 - Patient presents to ER with epigastric pain. He is stable. Ultrasound showed gallstones
with dilated CBD. Labs showed high amylase of 250.
What is the most appropriate next step?

A - ERCP
B - CT Abdomen
C - Cholecystectomy now
D - Cholecystectomy after 6 months

Answer:
High amylase “may” indicate acute pancreatitis (though you need amylase to be minimum of
three times upper normal value).

Dilated CBD “may” indicate obstructive jaundice. We don’t have bilirubin level though.

In general, mild, and moderately severely acute biliary pancreatitis are treated conservatively,
followed by laparoscopic cholecystectomy in a couple of days once patient’s symptoms improve.

Obstructive jaundice is treated by biliary decompression and ductal clearance via ERCP,
followed by laparoscopic cholecystectomy in the same admission.

_____________________________________

Q19 - Patient with colon polyp (Pedunculated polyp 1.5 cm). Colonoscopy in?

A - 1 year
B - 3 years
C - 10 years
D - Do nothing

Answer: Incomplete question

Other recalls were discussed previously where the polyp was removed, and the question is about
the surveillance afterwards.
Histopathology can be adenomatous polyp with or without dyspepsia, carcinoma in situ or
invasive adenocarcinoma.
Surveillance protocol depends on histopathology.

_____________________________________

154
Q20 - Patient with a history of sleeve bypass and history of leak. presents with severe pain and
mild hypotension.
What's your next step?

A - Endoscopy
B - US
C - Laparotomy
D - Laparoscopy

Answer:
This question is a bad recall.
First, there is no procedure called sleeve bypass. These are two different procedures. It is either
this or that.

Second, “with a history of leak”…


How was it managed?! Sealed or not?!!

Third, what is the timeframe of this recent presentation?!! How long after surgery?!!

_____________________________________

Q21 - Patient after bypass 4 months comes to ER with abdominal tenderness.


What to do next?

A - Endoscopy
B - Barium
(No CT or US in the options)

Answer: Incomplete scenario


Patients don’t come with “tenderness” they rather come with “pain” and pain got to have some
characteristics that points towards the likely diagnosis.

_____________________________________

Q22 - Female in her 60s comes to ER with acute abdomen that became more severe over the last
few hours. US, solitary cystic complex adrenal mass 5 x 6 cm. What to do?

A - MRI pelvic
B - Laparotomy
C - Observation

Answer:
Adrenal masses - even malignant ones - don’t present with “acute abdomen”.
_____________________________________

155
Q23 - After herniotomy surgery for 5-year-old boy came with fever and pus discharge and part
of mesh seen?

A - Give IV antibiotics
B - Draining of pus
C - Draining of pus and remove mesh
D - Observation

Answer:
Herniotomy - by definition - involves dividing the hernia sac only. No mesh is used.

_____________________________________

156

You might also like